#35 Rosh Review

Réussis tes devoirs et examens dès maintenant avec Quizwiz!

Question: What is the management for acid or base ingestions?

Answer: Upper endoscopy. Rapid Review Foreign Body Aspiration: Right main bronchus CXR: unilateral obstructive emphysema, edge of coin seen if in trachea Bronchoscopy

Question: What are some other menopause symptoms that are attributed to declining estradiol levels?

Answer: Vaginal atrophy/dryness, depression, memory difficulties and migraines. Rapid Review Menopause Decreased ovarian follicles --> decreased estrogen Average age: 51 Menopause before 40: premature ovarian failure Increased FSH is specific HAVOCS: Hot flashes, Atrophy of Vagina, Osteoporosis, CAD, Sleep abnormalities Vaginal bleeding: r/o malignancy

Question: Can amiodarone be used in patients with heart failure?

Answer: Yes, one of the attractive features of using amiodarone over other antidysrhythmics.

Question: How do normal vital signs change during pregnancy?

Answer: Heart rate increases by 10-15 beats per minute. Blood pressure typically drops during the 2nd trimester. Rapid Review Normal Pregnancy ↑: Blood volume, cardiac output, tidal volume ↓: Functional residual capacity, systemic vascular resistance Respiratory alkalosis: 3rd trimester Doppler heart tone at 10 weeks HR: ↑ 10-15 bpm BP: ↓ in 2nd trimester, normalizes in 3rd Serum ß-hCG: doubles every 2 days in early pregnancy Fundal height: 12 weeks: pubic symphysis 20 weeks: umbilicus 20-32 weeks: height (cm) above symphysis = gestational age (weeks) Transvaginal ultrasound: IUP visualized when ß-hCG > 1500 Transabdominal ultrasound: IUP visualized when ß-hCG > 4000

Question: What infection is the most common precipitator of erythema multiforme?

Answer: Herpes simplex virus. Rapid Review Lyme Disease NE USA Campers/hikers Ixodes tick harbors Borrelia burgdorferi Stage I: erythema migrans (pathognomonic), viral-like syndrome Stage II: arthritis, myocarditis, bilateral Bell's palsy Stage III: chronic arthritis, chronic encephalopathy Rx: doxycycline CNS/cardiac involvement: IV ceftriaxone Pregnant/children rx: amoxicillin

Question: What symptom is pathognomonic for rabies infection?

Answer: Hydrophobia. Rapid Review Rabies Patient with a history of exposure to raccoons, bats, or skunks Complaining of hydrophobia, agitation, spasms Treatment is wound care (scrubbing), Ig at wound site, vaccination

Question: For every 100 mg/dL of glucose above 100 mg/dL, how much does the sodium level decrease by?

Answer: It decreases by 1.6 mEq/L. Rapid Review Diabetic Ketoacidosis Patient will be a diabetic With a history of Infection, Ischemia (cardiac, mesenteric), Infarction, Ignorance (poor control), Intoxication (FIVE I's) Complaining of abdominal pain, vomiting, and fatigue PE will show hyperglycemia and ketonemia Labs will show anion gap metabolic acidosis Treatment is IV fluids & insulin infusion Comments: Corrected sodium add 1.6 mEq/L for each 100mg/dL in serum glucose

Question: What is the Hunt and Hess Clinical Grading Scale?

Answer: It is a scoring scale for cerebral aneurysms and subarachnoid hemorrhage. The scale ranges from 0 (unruptured aneurysm) to 5 (deep coma, decerebrate posturing). Rapid Review Subarachnoid Hemorrhage Sudden onset, thunderclap headache Ruptured berry aneurysm Polycystic kidney disease Head CT/LP Xanthochromia Nimodipine

Question: What is a Still's murmur?

Answer: It is a short, vibrating, physiologic murmur which can be heard over the mid precordium in children in the absence of any other abnormality. This is a common and benign condition.

Question: What is the most common disaccharide deficiency?

Answer: Lactase deficiency.

Question: What imaging modality is the gold standard for diagnosing an anterior cruciate ligament injury?

Answer: MRI. Rapid Review Patellofemoral Pain Syndrome Patient will be a women Complaining of aching anterior knee pain, that is worse with loaded flexion (stair climbing, jumping, prolonged sitting) PE will show patellar crepitation Treatment is strengthen medial quadriceps, NSAIDs

Question: What diagnostic test should be ordered in a patient with vertigo concerning for cerebellar infarction?

Answer: MRI/MRA of the brain is required as CT scan of the head does not visualize the posterior fossae well. Rapid Review Peripheral Vertigo Disorder of external/middle/inner ear of CN VIII Causes majority of vertigo Sudden onset Intermittent episodes of disequilibrium/spinning Hearing loss and tinnitus common Spontaneous nystagmus decreased by visual fixation Positional nystagmus fatigues and adapts Many etiologies Rx: antihistamines, antiemetics and benzodiazepines

Question: What is the treatment for pertussis? https://www.youtube.com/watch?v=8QWdEwjBEBw

Answer: Macrolide antibiotics (e.g., erythromycin, clarithromycin, azithromycin). Prevention is achieved through administration of the pertussis vaccine DTaP in children < 7 years old, Tdap in children > 7 years and adults. Rapid Review Pertussis ("Whooping Cough") Whooping cough: expiratory cough, inspiratory whoop Child with incomplete immunization hx URI sx → paroxysmal stage → convalescence Macrolide

Question: What electrolyte deficiency is associated with Prinzmetal angina?

Answer: Magnesium deficiency. Rapid Review Prinzmetal's Angina Intermittent coronary artery vasospasm Risk factors: smoking, cocaine ​ST elevations Negative cardiac biomarkers Rx: nitrates, CCBs

Question: What is akathesia?

Answer: Motor restlessness characterized by the need to constantly be moving. Rapid Review Extrapyramidal Symptoms Typical > atypical antipsychotics Acute dystonia: muscle spasms, stiffness, oculogyric crisis, rx: benztropine, diphenhydramine Akathisia: restlessness, rx: benztropine, ßBs Bradykinesia: Parkinsonism, rx: benztropine Tardive dyskinesia: orofacial involuntary movements

Question: What is the syndrome called when parents create illness in their children?

Answer: Munchausen by proxy. Rapid Review Munchausen Syndrome Desires "sick role" Hx of multiple hospital admissions Willingness to undergo procedures

Question: What medications might you discourage your patients from using when they present with subconjunctival hemorrhage?

Answer: NSAIDs.

Question: What is the duration of action for naloxone?

Answer: Naloxone will reverse opiate effects for 1-2 hours. Rapid Review Opioid Withdrawal Flu-like illness Abdominal cramps Diarrhea Mydriasis Piloerection Yawning Rx: clonidine, antiemetics

Question: What is the typical work-up of nystagmus?

Answer: Neuroimaging and videonystagmography. Rapid Review Peripheral Vertigo CN VIII, vestibular apparatus Onset: sudden Hearing loss, tinnitus Positional Nystagmus: unidirectional, never vertical, fatigable, can be inhibited No neurologic sx

Question: Should you directly challenge a delusional patient about his or her false beliefs?

Answer: No, avoid directly challenging the patient but do not pretend to be in full acceptance. Rapid Review Delusions Fixed, false belief Persecutory most common Schizophrenia, bipolar

Question: Will administering oral vitamin B12 help to treat B12 deficiency in pernicious anemia?

Answer: No, intramuscular B12 should be administered. Oral will not be absorbed. Rapid Review Megaloblastic Anemia B12 deficiency: vegan, pernicious anemia Folate deficiency: alcoholic, antifolate therapy MCV > 100 Hypersegmented neutrophils B12 deficiency: neurologic findings

Question: What are the 3 most common complications of diverticulitis?

Answer: Obstruction, perforation, abscess. Rapid Review Diverticulitis Patient will be complaining of abdominal pain that is localized to the left lower quadrant, fever, nausea, vomiting, and a change in bowel habits PE will show localized guarding, rigidity, and rebound tenderness Diagnosis is made by CT scan Treatment is abx

Question: What is the "pseudo-kidney sign?"

Answer: On ultrasound, the intussuscepted segment may have an appearance similar to that of a kidney. Rapid Review Intussusception (Telescoping Bowel) Patient will be a child 5 months - 3 years old Complaining of colicky abdominal pain, vomiting and bloody stools (currant jelly) Diagnosis is made by ultrasound ("target sign") Most commonly caused by a tumor or meckel's diverticulum Treatment is air/contrast enema

Question: What is the most significant risk of untreated idiopathic intracranial hypertension?

Answer: Optic atrophy and visual loss can result from untreated pseudotumor cerebri, so serial visual function testing is an essential part of management of this disease.

Question: Which medication provides systemic stimulation for patients with Sjögren's syndrome?

Answer: Pilocarpine. Rapid Review Sjogren's Syndrome Patient will be a woman 40 - 60 years old Complaining of dry eyes (Xerophthalmia) and dry mouth (Xerostomia) Labs will show SSA (anti-Ro) or SSB (anti-La) Diagnosis is made by positive Schirmer test Most commonly caused by autoimmune disorder

Question: What other tick-borne disease presents as a faint maculopapular rash that begins on the wrists and ankles and spreads to the extremities and trunk?

Answer: Rocky Mountain spotted fever (Rickettsia rickettsi) Rapid Review Lyme Disease NE USA Campers/hikers Ixodes tick harbors Borrelia burgdorferi Stage I: erythema migrans (pathognomonic), viral-like syndrome Stage II: arthritis, myocarditis, bilateral Bell's palsy Stage III: chronic arthritis, chronic encephalopathy Rx: doxycycline CNS/cardiac involvement: IV ceftriaxone Pregnant/children rx: amoxicillin

Question: What medication is used to ion trap aspirin in the urine thus enhancing elimination?

Answer: Sodium bicarbonate. Rapid Review Salicylate Toxicity Aspirin, wintergreen, Bismuth subsalicylate Respiratory alkalosis + anion gap metabolic acidosis Hypoglycemia Tinnitus Rx: AC (if < 1 hour from ingestion), urine alkalinization, K+ Hemodialysis indications: Level > 100 mg/dL (acute) or > 50 mg/dL (chronic) Coma Rising levels despite alkalinization Renal failure Pulmonary edema Altered mental status Clinical deterioration

Question: What is stasis dermatitis?

Answer: Stasis dermatitis occurs with venous insufficiency and valvular incompetency. The proximal skin appears thin and brown, and may occur with distal macules, papules, red irritation, skin thickening and edema. Rapid Review Varicose Veins Patient will be a woman Complaining of a dull ache in her legs after prolonged standing PE will show dilated, elongated subcutaneous veins Diagnosis is made by duplex ultrasound Treatment is leg elevation and compression stockings

Question: What are some complications of celiac disease if left untreated?

Answer: T-cell lymphoma and small intestine adenocarcinoma. Rapid Review Celiac Disease Patient will be complaining of diarrhea, steatorrhea, flatulence, weight loss, weakness and abdominal distension Labs will show IgA anti-endomysial (AGA) and anti-tissue transglutaminase (anti-tTG) antibodies Diagnosis is made by small bowel biopsy Treatment is gluten free diet Comments: associated with dermatitis herpetiformis (chronic, very itchy skin rash made up of bumps and blisters)

Question: What is the equation for the target heart rate used for an exercise stress test?

Answer: Target heat rate = 85 % (220 - age (in years)).

Question: At what age is the physiologic nadir for hemoglobin?

Answer: The lowest normal hemoglobin in a child occurs at three months of age, with a normal hemoglobin of 12.0 g/dL. Rapid Review Iron deficiency anemia: Most common cause of childhood nutritional deficit anemia Can be severe (hemoglobin 1-2 g/dL) but rarely fatal Children drinking only cow's milk Dietary history is the key to diagnosis Treat with oral ferrous sulfate, see response in 72 hours Treatment must include diet restructuring Can progress to congestion heart failure, particularly in setting of secondary insult (fever) Transfuse red blood cells in small aliquots, only if patient in heart failure or extremis

Question: Which aspect of the tympanic is most often involved in a cholesteatoma?

Answer: The pars flaccida (superior aspect of the TM). Rapid Review Acquired Cholesteatoma: Patient will have a history of chronic ear infections or tympanostomy tubes Complaining of painless otorrhea PE will show yellow or white mass behind the tympanic membrane Treatment is tympanomastoid surgery

A patient with an intermediate risk of coronary artery disease is undergoing an exercise stress test. Which of the following is the most specific finding for myocardial ischemia? 2 mm downsloping ST-segment depression 2 mm upsloping ST-segment depression Increase of systolic blood pressure Sporadic premature ventricular complexes

Correct Answer ( A ) Explanation: 2 mm downsloping ST-segment depression is the most specific finding for myocardial ischemia during an exercise stress test. Subendocardial ischemia during exercise produces ST-segment depression or elevation or both. ST-segment depression that occurs during exercise testing is one of the most identifiable ECG signs of myocardial ischemia. The ECG portion of the exercise test is generally considered abnormal, or positive for ischemia, when there is ≥ 1 mm horizontal or downsloping ST-segment depression in one or more leads. Horizontal or downsloping ST-segment depression is generally more specific for ischemia than upsloping ST-segment depression. An increase in systolic blood pressure (C) is an expected physiologic response during an exercise stress test and is not a sign of myocardial ischemia. Sporadic premature ventricular complexes (D) are not uncommon during peak exercise and do not indicate myocardial ischemia. Although 2 mm upsloping ST-segment depression (B) could be suggestive of myocardial ischemia, it is less specific than downsloping or horizontal ST-segment depression.

Which of the following wounds is a potential candidate for primary closure? Facial laceration from a dog bite Laceration on sole of the foot after stepping on an unknown object in a stream Laceration over the metacarpal-phalangeal joint sustained during a fistfight Puncture wounds on the leg as a result of a dog bite

Correct Answer ( A ) Explanation: Although dog bite wounds are high risk for infectious complications, those that are cosmetically deforming are often closed primarily, especially on the face. Dog bites are high risk for infection from organisms commonly found in the dog's mouth, which include S. aureus, P. multocida, and S. viridans. Although controversy exists over management of animal bite wounds, generally it is agreed that wounds that are cosmetically deforming (including those on the face) should be repaired primarily. All bite wounds must be thoroughly irrigated and cleansed. Wounds on the extremities and that are not as cosmetically concerning are generally left open or loosely approximated. Certain lacerations are best managed when left open or closed in a delayed fashion. These include lacerations that are heavily contaminated or have extensive associated tissue damage, human bites, animal bites that are not cosmetically significant, and contaminated lacerations on the bottom of the feet. A laceration on the plantar aspect of the foot sustained on an unknown object while walking in a stream (B) would be considered contaminated. Lacerations over the MCP joints (C) that occur during a fistfight are considered to be human bites and should not be closed primarily. Puncture wounds on the leg (D) from a dog bite are considered high risk for infection and are not cosmetically significant, therefore should not be closed primarily.

Question: What are potential common side effects of metformin?

Answer: Gastrointestinal side effects such a diarrhea and abdominal cramps. Less commonly, metformin causes lactic acidosis. Rapid Review Polycystic Ovarian Syndrome (PCOS) Patient will be an obese female Complaining of amenorrhea, obesity, hirsutism PE will show bilateral ovarian enlargement, acanthosis nigricans Labs will show high LH:FSH, androgen excess Most commonly caused by insulin resistance Treatment is combination OCP's, lifestyle changes, metformin

Question: Why do H2 blockers help with cutaneous symptoms in allergic reactions?

Answer: 10% of histamine receptors in the skin are of the H2 variety. Rapid Review Anaphylactic Reaction IgE-mast cell mediated Airway management Antihistamines, dexamethasone, IM epinephrine, IVF Glucagon for refractory hypotension in patient with known HTN

Question: What duration of CPR constitutes a relative contraindication to fibrinolytic use for STEMI?

Answer: > 10 minutes. Rapid Review Acute Coronary Syndrome: Management Aspirin: ↓ mortality, ↓ infarct size, ↓ reinfarction rate Clopridogrel: patients with aspirin allergy Heparin: ↓ DVT, ↓ reinfarction, ↓ stroke, ↓ LV thrombus, ↓ reocclusion Nitroglycerin: Coronary artery dilation/vascular smooth muscle relaxation → ↓preload/afterload → ↓ myocardial O2 demand Contraindications: sildenafil use within 24 hrs, RV infarction ß-blockers: ↓ Myocardial O2 demand, ↓ ventricular fibrillation IV indications: tachydysrhythmias, intractable HTN Morphine: ↓ Preload/afterload, ↓ sympathetic activity No mortality benefit Glycoprotein IIb/IIIa inhibitors: benefit in patients undergoing PCI PCI: Preferred over thrombolytics in all STEMI patients PCI center: <90 minutes contact to device time Non-PCI center: transfer to PCI center if contact to device time can be <120 minutes Non-PCI center: thrombolytics if contact to device time to be >120 minutes Thrombolytics: begin within 30 minutes of ED arrival if selected

Question: What is the most important predictor of mortality in a patient with necrotizing fasciitis?

Answer: A history of diabetes. Rapid Review Necrotizing Fasciitis Type 1: polymicrobial - abdomen and perineum Type 2: GAS infection - extremity Pain out of proportion to exam Systemic signs of infection Cellulitis turns dusky blue with bullae/vesicles Radiograph: subcutaenous emphyesma Emergent surgical debridement

Question: What is a Maisonneuve fracture?

Answer: A spiral fracture of the proximal 1/3 of the fibula associated with a tear of the distal tibiofibular syndesmsosis and the interosseous membrane. Rapid Review Ankle Sprain Partial or complete tearing of ≥ 1 ligaments MC due to inversion mechanism MC sprained: anterior talofibular ligament (ATFL) Grade I: microscopic tear, grade II: incomplete tear, grade III: complete tear RICE

Question: What is Miller-Fischer syndrome?

Answer: A variant of GBS characterized by opthalmoplegia and ataxia. Rapid Review Gullain-Barre Syndrome Patient with a history of recent minor respiratory or GI illness Complaining of symmetric, progressive ascending muscle weakness PE will show lack of deep tendon reflexes Lumbar puncture results will demonstrate increased CSF protein but a normal cell count Most commonly caused by Campylobacter jejuni Treatment is supportive, plasmapheresis or IVIG

Question: What agents should be avoided in patients with atrial fibrillation and Wolf-Parkinson-White (WPW) syndrome?

Answer: AV-nodal blocking agents such as adenosine, calcium channel blockers, beta-adrenergic blockers, and digoxin. This can lead to cardiovascular collapse due to preferential accessory pathway conduction. Rapid Review Atrial Fibrillation Alcohol Irregularly irregular No P waves Narrow QRS unless conduction block or accessory pathway Unstable: cardioversion Stable: Rate control with CCBs, ßBs <48 hours duration: cardiovert to sinus rhythm >48 hours duration: anticoagulate, echo to r/o thrombus, then cardioversion

Question: What medication can be used in hyperuricemia to decrease production of uric acid?

Answer: Allopurinol. Rapid Review Tumor Lysis Syndrome 1-5 days after chemotherapy Hematologic malignancy Hyperuricemia Hyperkalemia Hyperphosphatemia Hypocalcemia IVF

Question: What antibiotic is first-line therapy for human bite infection prophylaxis?

Answer: Amoxicillin-clavulanate. Rapid Review Dog Bites Crush injuries Gangrenous wound, sepsis: C. canimorsus Amoxicillin/clavulanate

Question: Which enzyme is elevated in over 75% of patients with sarcoidosis?

Answer: Angiotensin converting enzyme.

Question: In patients with a myocardial infarction, which three drugs have been shown to decrease mortality?

Answer: Aspirin, beta-blockers, and ACE-inhibitors. Rapid Review Acute Coronary Syndrome: Management Aspirin: ↓ mortality, ↓ infarct size, ↓ reinfarction rate Clopridogrel: patients with aspirin allergy Heparin: ↓ DVT, ↓ reinfarction, ↓ stroke, ↓ LV thrombus, ↓ reocclusion Nitroglycerin: Coronary artery dilation/vascular smooth muscle relaxation → ↓preload/afterload → ↓ myocardial O2 demand Contraindications: sildenafil use within 24 hrs, RV infarction ß-blockers: ↓ Myocardial O2 demand, ↓ ventricular fibrillation IV indications: tachydysrhythmias, intractable HTN Morphine: ↓ Preload/afterload, ↓ sympathetic activity No mortality benefit Glycoprotein IIb/IIIa inhibitors: benefit in patients undergoing PCI PCI: Preferred over thrombolytics in all STEMI patients PCI center: <90 minutes contact to device time Non-PCI center: transfer to PCI center if contact to device time can be <120 minutes Non-PCI center: thrombolytics if contact to device time to be >120 minutes Thrombolytics: begin within 30 minutes of ED arrival if selected Beyond The Boards The Air Versus Oxygen in ST-elevation Myocardial Infarction (AVOID) trial compared supplemental oxygen vs no oxygen unless oxygen fell below 94%. The AVOID study found in patients with ST-elevation myocardial infarction who were not hypoxic, administration of oxygen may increase myocardial injury, recurrent myocardial infarction, and major cardiac arrhythmia and may be associated with greater infarct size at 6 months.

Question: What is the typical distribution of weakness caused by West Nile meningoencephalitis?

Answer: Asymmetric; isolated facial involvement may also be seen. Rapid Review West Nile Virus Mosquitos Summer/fall Flulike sx, URI sx, rash Complication: meningoencephalitis

Question: Which tachydysrhythmia is most commonly seen in Wolff-Parkinson-White syndrome?

Answer: Atrioventricular reentrant tachycardia (AVRT).

Question: What is the age distribution of osteosarcoma?

Answer: Bimodal (peaks in adolescence and adults over 65 years).

Question: What is the preferred form of management in patients with sarcoidosis?

Answer: Corticosteroids (eg, prednisone).

Question: A patent ductus arteriosus is actually beneficial in infants with which other congenital heart diseases?

Answer: Ductal dependent lesions include transposition of the great vessels, Tetralogy of Fallot, tricuspid atresia, hypoplastic left heart and severe coarctation of the aorta. Rapid Review Patent Ductus Arteriosus Patient will be complaining of failure to thrive, poor feeding, tachycardia, and tachypnea PE will show continuous, rough, "machinery-like" murmur, heard best in the first interspaces of the LSB Diagnosis is made by echo Treatment is indomethacin

Question: What are the alarm symptoms for gastrointestinal disorders that require evaluation with esophagogastroduodenoscopy (EGD)?

Answer: Dysphagia, odynophagia, weight loss, blood in stool, anemia, age over 50, and chronic symptoms without a diagnosis.

Question: In patients identified as high risk for spontaneous bacterial peritonitis, which antibiotics are used prophylactically?

Answer: Fluoroquinolones or TMP-SMX. Rapid Review Spontaneous Bacterial Peritonitis Patient will have a history of chronic liver disease or cirrhosis Complaining of fever, chills, and abdominal pain PE will show ascites, shifting dullness Labs will show PMNs > 250, WBC >1,000, pH <7.34 Diagnosis is made by analysis of the ascitic fluid Most commonly caused by E. coli, Streptococcus spp Treatment is immediate IV antibiotics (third-generation cephalosporin)

A five-year-old boy is playing with his two-year-old sister when he calls to tell his mother that the girl put a dime in her mouth. The mother could not find anything in the girl's mouth so she brings her to the emergency department. Your physical examination is normal. Which of the following is the next best step? Obtain X-ray of the neck, chest, and abdomen Order barium contrast studies Provide reassurance to the mother Request for urgent endoscopy

Correct Answer ( A ) Explanation: Any history of foreign body ingestion should be taken seriously and investigated. The majority (80 percent) of foreign body ingestions occur in children between six months and three years of age. Coins and small toy items are the most commonly ingested foreign bodies. There may be symptoms of choking, gagging, and coughing that may be followed by excessive salivation, dysphagia, food refusal, emesis, or pain in the neck, throat, or sternal notch regions. There may be respiratory symptoms like stridor, wheezing, cyanosis, or dyspnea that may be encountered if the esophageal foreign body impinges on the larynx or the membranous posterior tracheal wall. Symptoms of cervical swelling, erythema, or subcutaneous crepitations suggest perforation of the oropharynx or proximal esophagus. The evaluation of a child with a history of foreign body ingestion should begin with plain anteroposterior (AP) radiographs of the neck, chest, and abdomen, along with lateral views of the neck and chest. Ordering barium contrast studies (B) may be helpful in an occasional asymptomatic patient with negative plain films. However, their use is discouraged because of the potential for aspiration and in making subsequent visualization and removal of the object more difficult. Providing reassurance to the mother (C) is not appropriate because the patient needs further investigation. Requesting an urgent endoscopy (D) may be the next step after plain films are obtained. Urgent endoscopy is not needed if plain films fail to visualize the object and the patient remains asymptomatic.

Which of the following is the most common cause of hypercalcemia in hospitalized patients? Malignancy Primary hyperparathyroidism Sarcoidosis Vitamin D excess

Correct Answer ( A ) Explanation: Hypercalcemia is usually the product of another underlying disorder and not a primary process in itself. Malignancy (A) is the most common cause of hypercalcemia in hospitalized patients. It is most closely associated with primary hematologic cancers, metastasis to bone, and parathyroid producing tumors. In a patient with known cancer, hypercalcemia is a poor prognostic factor. Symptoms include lethargy and weakness, constipation and anorexia. The mainstay in management includes hydration. Adjunctive therapy includes furosemide, bisphosphonates, and hydrocortisone. Primary hyperparathyroidism (B) is usually caused by a benign adenoma of the parathyroid gland. This leads to elevated parathyroid hormone secretion which leads to increased serum calcium via various mechanisms. Primary hyperparathyroidism is the most common cause of hypercalcemia in non-hospitalized patients. Sarcoidosis (C) is a multisystem granulomatous disorder of unknown etiology. Typical chest X-ray findings are bilateral hilar adenoapthy and reticular opacities. Hypercalcemia is commonly present in sarcoidosis as well as other granulomatous diseases such as tuberculosis due to Extrarenal synthesis of calcitriol [1,25(OH)2D3]. Patients can also present with various skin, joint, and eye lesions. Common presenting symptoms are cough, dyspnea, malaise, fever, and weight loss. The characteristic lesion on lung biopsy is a non-caseating granuloma. Vitamin D excess (D) is a rare cause of hypercalcemia. It is usually seen in patients with chronic kidney disease who are taking high doses of vitamin D on a daily basis.

A 54-year-old woman presents with chest and neck flushing. She states this began 6 months ago. The "warm feeling", as she describes it, is becoming more frequent, and seems to occur mainly in the evening. She denies any recent infections. There are no dermatologic lesions seen on physical examination. Her final menstrual period was 14 months ago. These symptoms are most likely due to a decline in which of the following hormones? Estradiol Follicle-stimulating hormone Progesterone Prolactin

Correct Answer ( A ) Explanation: Menopause is the end of female fertility and can be defined as amenorrhea for greater than 12 months. Age of onset is the early 50s, with an average age of 52-years-old. As the female body ages, the total number of follicles, which contain ova, decreases. As such, the ovaries become less sensitive to two pituitary hormones, luteinizing hormone and follicle-stimulating hormone. This leads to the inability to produce estrogens (namely estradiol) and progesterone, which results in an inability to regenerate and maintain the uterine epithelium. The overall effect is eventual amenorrhea, declining estradiol and progesterone levels and increased follicle-stimulating hormone levels (which is due to the loss of estradiol's negative feedback on the pituitary gland). The vasomotor instability symptoms of "hot flashes" are felt to be due to this abrupt decline in circulating estradiol. These symptoms are characterized by a chest, neck and facial flushing/warmth which occurs intermittently and mainly in the later hours of the day, which can lead to significant fatigue due to sleep interruption. Progesterone (C) does decrease in menopause, however, its decline is not felt to be causative of vasomotor instability. Rather, there is knowledge that suggests decreased estrogens are the primary cause of hot flashes. Prolactin (D) is an anterior pituitary hormone responsible for breast milk secretion. Menopausal vasomotor instability is not caused by a lack of this hormone. Follicle-stimulating hormone (B) increases during menopause.

Which of the following findings is most likely to occur with acute diverticulitis? Constipation Gross lower GI bleeding Right lower quadrant tenderness Testicular pain

Correct Answer ( A ) Explanation: The clinical signs and symptoms of acute diverticulitis vary with the severity of the disease. Steady pain is the most common symptom in acute diverticulitis, not colicky pain. Most cases in the United States start with vague and generalized pain that localizes to the left lower quadrant. Diffuse tenderness is associated with perforation or abscess formation. Bowel habits are commonly altered with diverticulitis, with constipation being more common than diarrhea, although an alternation between the two is also common. Urinary symptoms can occur due to intestinal inflammation near the ureter or bladder. Although stool samples may be guaiac positive for occult blood, gross bleeding is unusual. Gross lower GI bleeding (B) is associated with diverticulosis, which is the most common cause of painless lower GI bleeding in elderly patients. In the Western world, 85% of diverticular disease occurs at the sigmoid colon (left side) (C). However, in Eastern cultures, such as the Japanese, right-sided disease is more common. This difference is thought to be due to the different amounts of fiber in Western (less) and Eastern (more) diets. The pain of diverticulitis is a steady and deep pain. Testicular pain (D) is commonly seen with kidney stones and associated with colicky pain.

A 54-year-old man with cirrhosis presents for evaluation of abdominal pain. The pain is diffuse throughout the abdomen and associated with subjective fever at home. He has no vomiting, diarrhea or change in mental status. His vital signs are T 100.6°F, HR 102, BP 140/88, RR 12, and oxygen saturation of 100% on room air. Bedside ultrasound demonstrates ascites. Which of the following is an indication for intravenous antibiotics? Ascitic fluid neutrophil count of 300 cells/mm3 Ascitic fluid pH of 7.35 AST of 340 mmol/L Peripheral white blood cell count of 15,000 cells/mm3

Correct Answer ( A ) Explanation: The patient's presentation is concerning for spontaneous bacterial peritonitis (SBP), an acute bacterial infection in the ascitic fluid of patients with ascites in the setting of liver disease. Most commonly, gram negative enteric organisms are responsible for the infection and the treatment of choice is an intravenous third generation cephalosporin. Diagnosis is made based on an ascitic neutrophil count > 250 cells. Other test results have been correlated with SBP, but treatment is guided based on the neutrophil count of the ascitic fluid. An ascitic pH of 7.35 (B) is nonspecific for this illness. A pH less than 7.34 or a gradient of more than 0.10 between the arterial and ascitic pH may be an earlier indicator of early SBP. An AST of 340 (C) is not predictive of SBP. Patients with cirrhosis will often have elevated transaminases, and particularly alcoholic patients will have an AST higher than ALT. The peripheral white blood cell count of 15,000 (D) may indicate the presence of infection, but is not specific and does not mandate the initiation of intravenous antibiotic therapy.

A 68-year-old woman presents to the ED with chest pain. It occurs at rest, and has been getting worse over the past 2 hours. Her past medical history includes COPD, GERD, diabetes, urinary incontinence and Factor V Leiden. Her past surgical history is significant for carpal tunnel release four years ago and total hip arthroplasty 18 months ago. Blood pressure is 168/118 mm Hg, heart rate is 100 BPM and oxygen saturation is 95% on room air. An electrocardiogram reveals ST-elevation. An initial cardiac panel shows a positive troponin level. In this scenario, which of the following in this patient is a relative contraindication to fibrinolytic therapy? Diastolic blood pressure Factor V Leiden Gastroesophageal reflux disease Total hip arthroplasty

Correct Answer ( A ) Explanation: These symptoms are concordant with acute coronary syndrome, notably, acute ST-elevation myocardial infarction. While percutaneous coronary intervention is recommended, it is not always possible, due to facility abilities and services. In addition, fibrinolytic therapy is recommended, as ST elevation is indicative of complete coronary thrombotic obstruction. There are however absolute and relative contraindications to this type of therapy as the risk of internal hemorrhage could produce catastrophic events. Absolute contraindications include a history of any intracranial hemorrhage, cerebral vascular structural lesions or intracranial neoplasm. Others include ischemic stroke or head or facial trauma within the past 3 months and active bleeding or bleeding diathesis. Relative contraindications include systolic BP ≥ 180 or diastolic BP ≥ 110 mm Hg, as well as uncontrolled chronic hypertension, pregnancy and anticoagulant use. This patient's DBP is 118 mm Hg, above the recommended 100 mm Hg. Bleeding diatheses (tendency or predisposition), such as von Willebrand's disease and Factor V deficiency, are absolute contraindications to fibrinolytics. Factor V Leiden (B) is a pro-thrombotic coagulopathy, not a bleeding diathesis. As such, this is not a contraindication. Active peptic ulcer disease, not GERD (C), is a relative contraindication to fibrinolytic therapy. Major surgery, such as total hip arthroplasty (D), is a relative contraindication if it occurred within 3 weeks of acute coronary symptom onset.

A 62-year-old man reports to the ED with new-onset, crushing, left-sided chest pain, radiating to the left arm that began suddenly 35 minutes prior to arrival. The patient has a history of hypertension, hypercholesterolemia, diabetes mellitus, and a 60-pack-year smoking history. His EMS ECG demonstrates ST-segment elevation in leads II, III, and aVF. In the ED, his vital signs are BP 135/75, HR 98, and RR 18. What is the most appropriate next step? Arrange for the patient to have an emergent stress test Call the cath lab emergently and prepare the patient for transport Give the patient nitroglycerin and draw labs, including troponins Place the patient on a cardiac monitor, give the patient oxygen if hypoxic and administer aspirin

Correct Answer ( D ) Explanation: This patient has an acute myocardial infarction (MI). An acute MI is clinically characterized by left-sided, substernal, chest pain (often described as an "elephant on my chest," tightness, or pressure rather than pain itself) that radiates down the left arm or left jaw, diaphoresis, nausea/vomiting, and shortness of breath. These symptoms are a result of myocardial death due to coronary vessel occlusion or vasospasm, often as a result of rupture of an atherosclerotic plaque. The definition of an acute MI, as described by the European Society for Cardiology and American College of Cardiology (ACC) is a rise and fall of a cardiac biomarker (troponin) in addition to clinical symptoms, ECG changes, or coronary artery changes as noted on an interventional level. Risk factors for an acute MI include hypertension, hypercholesterolemia, diabetes, tobacco, male, increased age, and family history. This patient has ST-segment changes in leads II, III, and aVF, correlating to an inferior wall MI. In a patient with an acute MI, the first step in management is to place the patient on a cardiac monitor to recognize any dysrhythmias, establish a peripheral IV, give oxygen if hypoxic, and administer aspirin. An ischemic right ventricle becomes preload dependent because it can no longer pump blood to the left side of the heart. Administering nitroglycerin (C) (a preload reducer) to a patient having ischemia of the right ventricle can lead to severe hypotension. Because the right coronary artery often supplies the inferior aspect of the heart and the right ventricle, whenever there is evidence of ST-segment elevation in the inferior leads (II, III, aVF), it is important to exclude a right ventricular infarct by obtaining "right-sided leads." This is performed by placing the precordial leads on the right side of the patient's chest instead of the left. Lead rV4 is thought to be the most sensitive in identifying a right ventricular infarct. Similarly, labs, including serial cardiac biomarkers (troponin) should be drawn; however, this is not the most appropriate next step. It is important to notify the cath lab (B) of the patient's need for percutaneous intervention, but the first priority are the ABCs and administering an aspirin. The cath lab should be notified immediately after this is performed. Although a stress test (A) would be useful in a patient with cardiac risk factors, this patient has active cardiac symptoms and the ACC/AHA guidelines recommend such testing be performed when patients are free of ischemic or heart failure symptoms for at least 8-12 hours. In addition, this patient has evidence for a STEMI and should undergo percutaneous intervention in the cath lab.

A 59-year-old man presents complaining of a severe headache for three days that came on gradually, and is diffuse in nature. His blood pressure is 205/105 mm Hg. All other vital signs are normal. His physical exam and basic laboratory workup are negative. His electrocardiogram is shown above. Which of the following antihypertensives is contraindicated in this patient? Atenolol Enalapril Hydralazine Hydrochlorothiazide

Correct Answer ( A ) Explanation: Wolff-Parkinson-White syndrome (WPW) is characterized by an accessory conduction pathway that bypasses the AV node, resulting in a shortened PR interval and upsloping QRS wave, known as a delta wave. If conduction through the AV node is slowed or disrupted, patients are at risk for an accelerated tachyarrhythmia and sudden cardiac death. Therefore, any medication that slows AV nodal conduction is contraindicated in WPW. These can be remembered by the mnemonic "ABCD"-- adenosine, beta-blockers, calcium channel blockers, and digoxin. Atenolol is a beta blocker and is therefore contraindicated. Enalapril (B), an angiotensin-converting enzyme inhibitor, hydralazine (C), an arteriolar smooth muscle relaxant, and hydrochlorothiazide (D), a thiazide diuretic, do not interfere with AV nodal conduction and are therefore safe for use in patients with Wolff-Parkinson-White syndrome.

A young woman complains of daily diarrhea without abdominal pain. She brings in a sample that appears to contain much more fat than water. She has a family history of diarrhea. She denies any recent antibiotic therapy. Her vital signs are within normal limits and her physical exam is unremarkable. Laboratory testing reveals iron deficiency anemia, elevated stool osmotic gap and positive anti-endomysial antibodies. Which of the following is the most likely diagnosis? Bile acid deficiency Celiac disease Clostridium difficile infection Irritable bowel syndrome

Correct Answer ( B ) Explanation: Celiac disease, originally known as celiac sprue, is an autoimmune disorder characterized by a malabsorptive state of chronic fatty diarrhea. It is caused by a genetic predisposition to an immune reaction against gliadin, a component of gluten, the main storage protein of wheat and similar grains. This immune reaction occurs in the lining of the small intestine, resulting in crypt hyperplasia and villous atrophy. These altered intestinal mechanics lead to malabsorption, which may lead to osteoporosis, iron deficiency anemia and elevated liver enzymes, as well as a high stool osmotic gap. Diagnosis requires a serologic search for specific markers, namely anti-endomysial and anti-tissue transglutaminase antibodies. Bile is necessary for fat absorption. A deficiency of bile acids (A) therefore may lead to fatty diarrhea. This is possible when it is not produced, as in liver disease like cirrhosis, or when it cannot pass into the intestines, as in the cholestasis diseases. This patient's history does not suggest either of these conditions. Clostridium difficile infection (C) can lead to an acute, watery diarrhea common in those on chemotherapy, antibiotics, the elderly and nursing home patients. This patient has no such history. Irritable bowel syndrome (D) is usually heralded by abdominal pain and a normal stool osmotic gap, of which this patient does not exhibit.

A 25-year-old man presents to your office after having noticed a slightly raised red lesion with central clearing that increased in size over the past five days. He describes feeling flu-like symptoms and states that he had been camping in Connecticut recently. Based on the clinical presentation, which of the following is the best treatment option for this patient? Ceftriaxone 250 mg single dose intramuscular injection Doxycycline 100 mg oral tablet twice per day for 21 days Metronidazole 2 g single dose tablet orally Watch and wait

Correct Answer ( B ) Explanation: Doxycycline 100 mg oral tablet twice per day for twenty-one days is the correct treatment and dosing for the suspected diagnosis of Lyme disease with a known erythema migrans (bulls-eye) rash. Lyme disease is caused by the spirochete Borrielia burgdorferi. It is transmitted to humans through a tick bite. The disease is most common in the northeastern United States, with Connecticut having one of the highest prevalence rates. It is recommended to treat any patient with suspected Lyme disease with an associated erythema migrans rash. Ceftriaxone 250 mg single dose intramuscular injection (A) is the correct treatment for urethritis caused by Neisseria gonorrhoeae. Gonococcal urethritis typically presents with painful urethral discharge. Patient's with urethritis should be treated concurrently for Chlamydial infection since the two infections commonly occur together. Metronidazole 2 g single dose orally (C) is the treatment for a trichamonal infection. Trichamonas is a flagellated protozoan, which infects the genitourinary tact of males and the lower urinary tract of females. Associated clinical findings include a malodorous, frothy, yellow-green discharge and a wet-mount that reveals motile flagellates. Watch and wait (D) is not the treatment of choice. Though it is suggested to send blood work for a confirmatory laboratory titer, the proper treatment course is to treat any patients with symptoms suggestive of Lyme disease. Delayed treatment can result in central nervous system and cardiac complications such as Bells' palsy, encephalitis, pericarditis, and heart block.

Which of the following is the characteristic rash associated with early Lyme disease? Erythema marginatum Erythema migrans Erythema multiforme Erythema nodosum

Correct Answer ( B ) Explanation: Erythema migrans is the primary manifestation of early Lyme disease. It usually occurs a few days to one month following an Ixodes tick bite. Typically, a red rash develops that expands gradually and often clears in the center, forming a ring. This is often referred to as a bull's eye rash. Erythema migrans may be accompanied by fever and regional lymphadenopathy and a variety of more subjective symptoms such as fatigue, malaise, lethargy, headache, neck stiffness, myalgias, and arthralgias. These flu-like signs and symptoms infrequently occur in the absence of erythema migrans. Erythema marginatum (A) is an evanescent, pink or faintly red, non-pruritic rash involving the trunk and sometimes the limbs that is associated with acute rheumatic fever. Erythema multiforme (C) is an acute, immune-mediated condition characterized by the appearance of distinctive target-like lesions on the skin. These lesions are often accompanied by erosions or bullae involving the oral, genital or ocular mucosae. It is most commonly induced by infection. Erythema nodosum (D) is characterized by red or violet subcutaneous nodules that usually develop in a pretibial location and is presumed to represent a delayed hypersensitivity reaction caused by various infectious agents, drugs, and other diseases.

Which of the following is true regarding intestinal intussusception? Adults are more affected than children Most adult intussusception cases involve the small bowel Most children with intussusception have a pathologic lesion The classic triad of abdominal pain, mass and heme-positive stools is usually seen in adults with intussusception

Correct Answer ( B ) Explanation: Intussusception in adults is rare but when present, involves the small bowel in 80% of cases. Intussusception involves telescoping of a proximal segment of the bowel into a distal segment. This process causes the mesenteric blood supply to the section of bowel to become compromised leading to ischemia, edema, infarction and possibly gangrene. In adults, intussusception typically presents with symptoms consistent with partial bowel obstruction. Vomiting, bleeding per rectum and constipation are common. The abdomen may become distended as well. As the disease progresses and bowel is compromised, sepsis may ensue. In adults, large bowel obstruction is frequently associated with malignancy. Although plain radiographs can be suggestive of the diagnosis, CT is the definitive study. Intussusception is more common in children than in adults (A). Most children with intussusception do not have a pathologic cause (C). The classic triad (D) is uncommon in adults as a mass is rarely palpated.

A 35-year-old woman is being evaluated at her annual well woman exam. She has no complaints and generally feels healthy. She has no past medical history and takes no medications. She walks 30 minutes five days per week with no dyspnea or discomfort. On physical exam her blood pressure is 118/68, pulse 64/min and respiration rate is 13/min. Her body mass index is 22. Cardiac auscultation reveals a 2/6 mid-systolic crescendo-decrescendo murmur heard best at the left lower sternal border without radiation. She has a normal S1 and S2 and normal cardiac impulse. Lungs are clear and peripheral pulses are normal. Electrocardiogram is normal. What is the next appropriate step in management? Cardiac magnetic resonance imaging No additional testing Transesophageal echocardiography Transthoracic echocardiography

Correct Answer ( B ) Explanation: No additional testing is indicated in this patient. She most likely has a benign systolic ejection murmur. This is a common clinical situation and reassurance is appropriate. A systolic murmur is present in up to 60 % of patients, with 90 % being associated with a normal echocardiogram. A murmur is characterized by its intensity or grade, timing, configuration, frequency or pitch, and location. The most common causes of a mid-systolic murmur are innocent flow murmurs, an increase in flow rate across a normal semilunar valve, and aortic valve sclerosis. Innocent murmurs are typically systolic ejection murmurs located at the base of the heart, short and soft, grade 1/6 to 2/6, without radiation, with a normal S1 and S2, normal cardiac impulse, and no evidence of any hemodynamic abnormality. The benign characteristics of the murmur in this patient include a grade < 3/6, mid-systolic timing, lack of radiation, and the absence of additional abnormal heart sounds. The remainder of the physical examination and the electrocardiogram are normal, without any evidence of cardiac enlargement or dysfunction. Signs of more serious cardiac disease include an S4, grade >3/6 intensity, any diastolic murmur and fixed splitting of S2. Transthoracic echocardiography (D) is the primary test for diagnosis and assessment of valvular heart disease. The appropriateness of this test is guided by history and physical examination. It is indicated in symptomatic patients, in those with a systolic murmur grade 3/6 or greater, diastolic murmurs, continuous murmurs (begins after s1 and beyond s2), holosystolic murmurs, late systolic murmurs, murmurs associated with ejection clicks, or murmurs that radiate to the neck or back. This patient's murmur does not have any of these characteristics. Transesophageal echocardiography (C) may be useful in patients with poor imaging by transthoracic study or to evaluate the feasibility of surgical repair when surgery is planned. Cardiac magnetic resonance imaging (A) is indicated if both transthoracic and transesophageal echocardiograms are equivocal in a more suspicious murmur. It is good for the assessment of the aorta and cardiac chamber size and function.

A continuous systolic-diastolic murmur is auscultated in a 3-week-old dyspneic infant. She is tachypneic and diaphoretic. Her mother reports weight loss and poor feeding. Doppler color-flow imaging reveals high velocity jets in the pulmonary artery. This patient will most likely undergo which of the following corrective surgeries? Arterial-switch Ligation Shunting Valvulotomy

Correct Answer ( B ) Explanation: Patent ductus arteriosus (PDA) is an acyanotic congenital cardiac condition. In this condition, the fetal connection between the pulmonary artery and aorta remains open, causing a continuous, "machine-like" murmur that is heard in both systole and diastole. Diagnosis is confirmed by appreciating pulmonary artery high-velocity jetting during Doppler echocardiography. Significant cases commonly result in labored breathing, symptoms of congestive heart failure and failure to thrive. Symptomatic PDA is treated surgically, with ligation being the most preferred method of closure. The arterial-switch (A) procedure is used to correct great artery transposition, not PDA. In tetralogy of Fallot, a shunt (C) is placed between the subclavian and adjacent pulmonary arteries. Shunts are not used in treating PDA. Valvulotomy (D) is used to correct aortic or pulmonary stenosis, two other common congenital cardiac defects.

An 18-year-old woman, who was just fired from her job, admits to ingesting 50 tablets of 325 mg strength aspirin three hours prior to arrival in the ED. She complains of dyspnea and tinnitus but denies any nausea and vomiting. On physical examination, you find a moderately distressed diaphoretic woman with tachycardia and tachypnea but clear lungs bilaterally and an oxygen saturation of 98%. You ask the nurse to initiate intravenous access, draw labs, and continuous cardiac monitoring. The arterial blood gas results are called back to you by the lab. What is the classic early blood gas seen in adults with acute aspirin ingestion? Metabolic acidosis Metabolic acidosis and respiratory alkalosis Metabolic and respiratory acidosis Triple acid/base disorder (respiratory alkalosis, high anion gap metabolic acidosis, and metabolic alkalosis)

Correct Answer ( B ) Explanation: Salicylates (aspirin) directly stimulate the respiratory center of the brain to cause hyperventilation and a subsequent respiratory alkalosis. A concurrent metabolic acidosis also develops, primarily from the uncoupling of oxidative phosphorylation, which leads to anaerobic metabolism, lactate production (with development of an anion gap), and hyperthermia. There is increased fatty acid metabolism as well, which results in excess ketone production and superimposed ketoacidosis. The respiratory alkalosis is not compensation for the metabolic acidosis. Isolated metabolic acidosis (A) may be seen with acute aspirin poisoning in children, but in adults, a mixed acid/base disorder is expected on blood gas analysis. Development of a metabolic and respiratory acidosis (C) may ensue in patients with apnea (perhaps from prolonged exposure to the toxic effects of aspirin), but it is not a feature of acute salicylate toxicity. A triple acid/base disorder (D) can be found in salicylate-poisoned patients with active vomiting, a clinical feature missing from this case.

A 45-year-old man who lives alone and is a highly functional working individual, tells you that the police are coming into his home each night and stealing food from his refrigerator and unlocking all the doors. Which of the following is the most likely diagnosis? Delirium Delusional disorder Depression Schizophrenia

Correct Answer ( B ) Explanation: The term delusional disorder refers to a condition whose core feature is persistent, nonbizarre delusions not explained by other psychotic disorders. It is a fixed false belief that has a certain level of plausibility. The delusion may emerge gradually and become chronic, and sometimes is associated with a precipitating event. Behavioral, emotional, and cognitive responses generally are appropriate, and neither mood disorders nor schizophrenic illness is present. There are several types of delusions, and the predominant type is identified to make the diagnosis. Minimal deterioration in personality or function and the relative absence of other psychopathologic symptoms have been considered important evidence for distinguishing this disorder from schizophrenia and other psychotic condition. Delirium (A) has a fluctuating course, with confusion, memory impairment, and transient delusions that contrast with the persistent delusions in most idiopathic delusional disorders. Profound changes in mood suggest depression (C). In delusional disorders, mood may be depressed, but the change usually is not as overwhelming and pervasive as in depression. Depression refers to a group of signs and symptoms, such as changes in appetite, sleep, libido, concentration, decisiveness, interest, and energy. Schizophrenia (D) should be considered when the delusions are bizarre; affect is blunted or incongruent with thinking; thought disorder, if present, is pervasive; and role functioning is impaired.

A mother brings in her 16-month-old boy because she is concerned that he appears pale. Vital signs are within normal limits for age except for a mild tachycardia. He appears pale on physical exam, is irritable but consoles appropriately, and appears tired. You hear a systolic flow murmur and tachycardia on cardiac exam, but otherwise, physical exam is normal. Laboratory testing reveals the following: What is the best treatment for this child's anemia? Oral ferritin supplementation Oral iron supplementation Packed red blood cell transfusion Parenteral iron transfusion

Correct Answer ( B ) Explanation: This is a case of severe iron deficiency anemia of childhood. This most commonly occurs in children between the ages of one and two years old and is often found in children who drink cow's milk exclusively for their nutritional intake. Pallor is usually noted when a patient's hemoglobin drops below 8 mg/dL. ​Ddecreased hemoglobin production, increased erythrocyte destruction, or blood losscan cause anemias of childhood. The most common cause of decreased hemoglobin or erythrocyte production in children is nutritional iron deficiency anemia. The patient's dietary history is the key to this diagnosis, and that may only be elucidated by careful questioning. This is a chronic disease that rarely needs acute parenteral treatment or transfusion, with a gradual worsening in pallor and symptoms. Treatment with oral iron is often initiated in the ED before confirmatory blood testing has been obtained. Occasionally, patients present to the ED when they are challenged by a fever or other acute illness. The increase in cardiac output, in a child with severe anemia, may be enough to cause the patient to go into congestive heart failure. Treatment for any child with severe anemia is not withheld if the child is in extremis. Treatment of iron deficiency anemia, as long as the patient is not in extremis or showing signs of congestive heart failure is oral elemental iron in the form of ferrous sulfate, 3 to 6 mg/kg/day. The medication should be given at night, on an empty stomach, in a single dose. The patient will begin to improve within 72 hours of treatment initiation. As much as iron supplementation is important in the treatment of iron deficiency anemia, it is also critical to restructure the diet of the child to assure adequate nutritional intake. Because improvement occurs very quickly after treatment initiation, it can be difficult to maintain follow-up with parents who no longer perceive their child as ill. Oral ferritin supplementation (A) is not the appropriate treatment for iron deficiency anemia. Packed red blood cell transfusion (C) is only appropriate if the patient is showing signs of extremis. This patient has an elevated heart rate and a systolic flow murmur, but is otherwise well and appropriate for oral therapy. If a child with severe iron deficiency anemia presents in congestive heart failure red blood cell transfusion may be necessary but should only be done in small amounts so as not to worsen the heart failure. Parenteral iron transfusion (D) carries with it a danger of anaphylaxis, and the hematologic response to iron supplementation is the same whether the treatment is administered orally or intravenously.

A 63-year-old man presents with dizziness. He states that when he turns his head to the right, he gets an intense sensation of room spinning with nausea and vomiting. The symptoms resolve in minutes with rest. His physical examination is remarkable for right-beating nystagmus when his head is turned to the right but is otherwise normal. What management is indicated? Antibiotics Epley maneuver Non-contrast head CT Prednisone

Correct Answer ( B ) Explanation: This patient presents with benign paroxysmal positional vertigo (BPPV), which can be cured by the Epley maneuver. True vertigo is defined as a sensation of disorientation along with a sensation of movement. It has a number of causes, which can be grouped into central and peripheral categories. Peripheral causes, while causing distressing symptoms, are generally benign and include BPPV, acute otitis media, labyrinthitis, Meniere's disease, vestibular neuronitis and trauma. Central causes of vertigo are usually more dangerous and potentially life threatening. These include infection (meningitis, encephalitis), vertebral basilar artery insufficiency, cerebellar hemorrhage or infarction, temporal lobe epilepsy and tumors. The table below delineates some of the different characteristics. BPPV presents with intermittent episodes of severe vertigo that are sudden in onset. The episodes often are triggered with specific head movements and resolve with time when not moving the head. In between episodes of vertigo, patients may have continued nausea. BPPV is caused by the presence of an otolith in one of the semicircular canals. Symptomatic treatment with antiemetics may be necessary depending on the severity of nausea. A Dix-Hallpike procedure can be performed on patients with vertigo to confirm the diagnosis. The Epley maneuver offers definitive treatment via otolith expulsion from the semicircular canal. Antibiotics (A) should be initiated in the treatment of suppurative labarynthitis or acute otitis media. Patients with suspected central causes of vertigo should have a non-contrast head CT (C) performed as part of their workup but this test plays no role in the diagnosis or management of peripheral vertigo. Prednisone (D) is useful in the treatment of vestibular neuronitis but not in BPPV.

A 23-year-old with acute lymphocytic leukemia (ALL) presents with generalized weakness and muscle cramps. The patient states that he started chemo 3 days ago. A basic metabolic panel shows serum potassium of 6.5 mEq/L, calcium of 6.3 mg/dl and a creatinine of 11.1 mg/dL. What management is indicated? Aggressive fluids and admit Aggressive fluids and emergent dialysis Kayexalate and admit Urine alkalinization

Correct Answer ( B ) Explanation: This patient presents with symptoms and signs of tumor lysis syndrome (TLS) and multiple electrolyte and metabolic abnormalities requiring dialysis. TLS occurs secondary to increased cell death in a rapidly growing tumor. It frequently occurs hours to a few days after the initiation of chemotherapy or radiation therapy. TLS is commonly seen in hematologic malignancies (ALL, non-Hodgkin's lymphomas) and in some solid tumors like small-cell lung carcinoma. Patients will present with symptoms and signs that reflect their electrolyte abnormalities and can be non-specific depending on severity. With hyperkalemia, ECG changes are common and can be exacerbated by concomitant hypocalcemia. Hyperphosphatemia and hyperuricemia are also common. Kidney function is a critical factor in the development and treatment of metabolic abnormalities as it is responsible for the removal and resorption of the various electrolytes involved. Initial therapy should be with stopping chemotherapy and/or radiation and starting fluids. Further care depends on the electrolyte abnormalities present. With severe metabolic derangements, hemodialysis is required. The figure below contains the six indications for emergent hemodialysis: While aggressive fluids are required (A) they are not adequate for treatment in this patient with elevated potassium and creatinine. Kayexalate (C) may treat hyperkalemia but is delayed in onset and will not treat the other metabolic derangements. Urine alkalinization (D) is indicated in hyperuricemia associated with tumor lysis syndrome and can be accomplished with acetazolamide.

Which of the following is associated with pernicious anemia? Homocysteine levels are decreased Methylmalonic acid is increased Normocytic erythrocytes Vitamin B12 elevation

Correct Answer ( B ) Explanation: Vitamin B12 is found in animal products and binds to intrinsic factor (IF) secreted by gastric parietal cells. This complex is absorbed in the terminal ileum. Pernicious anemia is an autoimmune disorder in which antibodies act against intrinsic factor and gastric parietal cells leading to chronic atrophic gastritis and decreased production and function of intrinsic factor. This subsequently leads to vitamin B12 deficiency. In vitamin B12 deficiency, serum methylmalonic acid is increased. In elderly patients, this form of megaloblastic anemia is one of the leading causes of vitamin B12 deficiency. Pernicious anemia is associated with other immunologic diseases such as Sjögren's syndrome, Hashimoto's disease, type 1 diabetes mellitus, and celiac disease. It is also associated with an increased risk for gastric cancer and carcinoid tumors. Vitamin B12 deficiency caused by dietary deficiency or malabsorption is rare. Dietary causes of deficiency are limited to elderly people who are already malnourished. Since the 1980s, the malabsorption of vitamin B12 has become rare, due to the decreasing frequency of gastrectomy and surgical resection of the terminal small intestine Other disorders associated with vitamin B12 malabsorption include deficiency in the exocrine function of the pancreas after chronic pancreatitis (usually alcoholic), lymphomas or tuberculosis of the intestine, Crohn's disease, Whipple's disease, and celiac disease. Uncommon etiology also includes nitrous oxide anesthesia and abuse. In vitamin B12 deficiency (<150 pmol/L), homocysteine levels are increased (A), the erythrocytes are usually macrocytic (MCV >100 fL) (C), serum vitamin B12 level is low (<200 pg/mL) (D). In folate deficiency, testing the red cell folate concentration is more reliable than the serum level.

A 14-year-old girl is in your clinic with her mother because of leg pain. She fell from her bike last month and hit her left leg. Since then, she has complained of pain on her left leg that waxes and wanes. On physical examination, you could palpate a soft tissue mass that is tender. Laboratory findings show elevated alkaline phosphatase and elevated lactate dehydrogenase. Radiograph reveals destruction of the normal trabecular bone pattern and a soft tissue mass that is ossified in a radial or sunburst pattern. Which of the following is the most likely diagnosis? Chondroblastoma Ewing sarcoma Osteomyelitis Osteosarcoma

Correct Answer ( D ) Explanation: "sunburst" pattern. Chondroblastoma (A) is a benign bone tumor and is characterized by a well-defined or sclerotic border, lack of destruction of the cortex, and lack of extension into the soft tissue on radiograph. Ewing sarcoma (B) is a neoplastic disease where radiographs show periosteal reaction that produces layers of reactive bone, deposited in an "onion peel" appearance. Osteomyelitis (C) is infection localized to bone and presents with systemic symptoms (fever) and local findings (tenderness, warmth, erythema, and swelling).

A 51-year-old man presents to the ED with foot pain. Vital signs are BP 105/80, HR 122, RR 18, and T 38.4°C. Finger stick blood glucose is 445 mg/dL. You obtain a radiograph of the foot as seen above. Which of the following is the most appropriate next step in management? Administer 2 L of normal saline and 10 units of regular insulin Administer parenteral antibiotics and perform an incision and drainage Begin antibiotics and consult general surgery Send ESR, CRP, and consult orthopedics

Correct Answer ( C ) Explanation: The radiograph reveals gas in the soft tissue of the foot. In conjunction with the patient's clinical presentation, the diagnosis is necrotizing fasciitis. Management includes surgical debridement by a surgeon and parenteral antibiotics. Supportive measures with intravenous fluids and possible vasopressor support may be indicated. Necrotizing fasciitis is more common in men than in women and frequently involves the lower extremities. There are 2 types of necrotizing fasciitis: Type 1 is polymicrobial and involves nongroup A streptococci plus anaerobes and usually occurs on the abdomen and perineum; Type 2 is due to group A beta-hemolytic streptococci, and the infection usually involves the extremities. Clinical findings include tenderness, edema, and erythema at the site of infection. Fever is usually present. Numbness or deep pain out of proportion to the exam is classic. The presentation can be deceiving because in many cases, the skin will appear normal, but the patient will complain of pain. By days 2 to 4, necrotic patches and bullae appear. At this point, gas may appear on the radiograph, but the absence of gas does not exclude the diagnosis. Due to the rapid spread of the infection, patients need to undergo immediate surgical debridement to remove the dead tissue. Broad-spectrum antibiotics and intravenous fluids should also be administered. Administering 2 L of normal saline and 10 units of regular insulin (A) is the standard treatment for acute hyperglycemia. The patient can benefit from fluid resuscitation and insulin; however, the most critical next step is to arrange for surgical debridement and begin broad-spectrum antibiotics. The greater the delay to starting antibiotics and debridement, the worse the prognosis. The patient requires far more than an incision and drainage (B). He will require significant debridement and possible amputation. The workup for osteomyelitis includes obtaining an ESR, CRP, and consulting orthopedics (D).

A 40-year-old woman with a history of asthma presents to the ED with symptoms of wheezing and shortness of breath similar to previous exacerbations. Her vital signs are BP 115/70, HR 80, RR, 14, and pulse oximetry is 99% on room air. The patient is offered and agrees to a point-of-care beta-hCG test that returns positive. On exam, you note mild bilateral wheezing with good air movement. Pelvic exam reveals a closed os without adnexal tenderness or masses. Which of the following is the most appropriate next step in management? Delay treating her asthma until her pregnancy status is further clarified Treat her asthma as indicated, and perform a beta-hCG quantitative level Treat her asthma as indicated, if improved, discharge with outpatient obstetrical follow-up Treat her asthma as indicated, perform a beta-hCG quantitative level, and obtain a pelvic ultrasound

Correct Answer ( C ) Explanation: As a good public health practice, many EDs offer routine screening for pregnancy using point-of-care testing. In this patient—with isolated respiratory complaints and lack of findings on physical or pelvic exam that indicate anything other than a normal pregnancy—an incidental positive beta-hCG does not warrant any further attention other than a referral for follow-up with an obstetrician. The patient's asthma is the treatment priority. Standard therapy should be administered even in the setting of pregnancy. Treatment should not be delayed (A). In fact, the most important aspect to assuring the health of the fetus is to ensure the health of the mother. No further workup of the patient's pregnancy is required (B and D). Her respiratory complaints are unrelated to her pregnancy.

A patient is diagnosed with Campylobacter jejuni enteritis, receives treatment and recovers. Two weeks later, he presents with weakness in his legs followed by weakness in his arms. He denies dyspnea, but reports diplopia and dysphagia. Examination reveals a mild fever, extraoccular muscle palsies, hyporeflexia, distal paresthesias and intact proprioception. Laboratory testing is significant for a normal CBC and a PaO2 of 88 mmHg. Which of the following is the most appropriate intervention at this time? Continuous positive airway pressure (CPAP) Corticosteroids Plasmapheresis Vitamin B12

Correct Answer ( C ) Explanation: Guillain-Barre syndrome (GBS) is a symmetric, progressive ascending muscle weakness that usually starts in the legs and may be acute or subacute. The condition is life-threatening if respiratory or swallowing muscles are involved. GBS can follow minor respiratory or GI illness, inoculation, or surgical procedures. It carries a poorer prognosis when it follows Campylobacter jejuni infection. The clinical hallmark of GBS is lack of deep tendon reflexes. Patients experience weakness of >2 limbs that typically begins with the proximal lower extremities. As weakness progresses, patients may experience shortness of breath, constipation, facial weakness, dysphagia, ophthalmoplegia, dysarthria. And sensory disturbances. Lumbar puncture results will demonstrate increased CSF protein but a normal cell count. Treatment is supportive. Plasmapheresis or IVIG may improve recovery time and decrease residual neurologic effects. CPAP (A), or other ventilatory assist device, is not recommended in a patient without dyspnea or with a normal arterial oxygen partial pressure. CPAP is more commonly used for obstructive sleep apnea than acute ventilatory support. Several controlled studies have shown corticosteroids (B) to be of no benefit. Vitamin B12 (D) deficiency causes a subacute or chronic syndrome of neurologic symptoms. It mainly affects the dorsal and lateral spinal cord columns leading to distal paresthesias, imbalance and altered vibrioception and proprioception. It occurs with macrocytic megaloblastic (pernicious) anemia. Chronic cases are associated with dementia. Folate deficiency can also present in a similar fashion with similar spinal cord pathology.

A 60-year-old woman presents with vertigo, tinnitus, hearing impairment and nystagmus. The nystagmus is unidirectional, with the fast phase directed to the right side. When asked to fixate on an object, the nystagmus lessens. Which of the following is the most likely diagnosis and the most likely body location of the neurological lesion. Central acquired vestibular nystagmus; lesion on the left side of the body Central acquired vestibular nystagmus; lesion on the right side of the body Peripheral acquired vestibular nystagmus; lesion on the left side of the body Peripheral acquired vestibular nystagmus; lesion on the right side of the body

Correct Answer ( C ) Explanation: Nystagmus is a condition of involuntary, periodic eye oscillation that can be physiologic, congenital or acquired, conjugate or disconjugate, and can occur in a horizontal, vertical or rotational plane (or combinations of these). The oscillations typically have a slow initiating phase followed by a fast corrective phase. Normally, eye position is maintained by interactions between three mechanisms: foveal centration (the oculomotor system keeps the object of interest focused on the fovea, thus giving the best acuity of that object), the vestibulo-ocular reflex (foveal centration is maintained during changing head movements via input form the vestibular system) and the neural integrator, otherwise known as the gaze-holding system (when the eye is positioned to an extreme in the orbit, the soft tissues want to "pull" it back to the center; this force is overcome by interactions between the cerebellum, vestibular system and oculomotor nuclei). Nystagmus results when there is an abnormality in any of these three mechanisms. One type of nystagmus is called vestibular nystagmus, which can be physiologic (watch someone's eyes as they spin in a chair) or pathologic (also called acquired), especially if the patient also complains of vertigo. Acquired vestibular nystagmus can be further defined as central (CAVN) (an abnormality exists in the CNS, i.e., brain or cerebellum) or peripheral (PAVN) (problem in the eight cranial nerve or the proprioceptors of the inner ear's semicircular canals). Typically the fast phase of nystagmus is directed towards (B & D) the lesion in central vertigo and opposite the lesion in peripheral vertigo. Typically, peripheral vertigo, not central vertigo (A & B) is associated with tinnitus and hearing impairment.

A 40-year-old woman complains of recurrent chest pain that occurs shortly after she wakes up in the morning. She has a history of migraine headaches and Raynaud's phenomenon. She admits to tobacco abuse and smokes 1 pack-per-day. Her chest pain is not reproducible with palpation. An ECG during an episode reveals ST-elevation in multiple leads and cardiac biomarkers are normal. Which of the following is the most likely diagnosis? Costochondritis Panic disorder Prinzmetal angina Unstable angina

Correct Answer ( C ) Explanation: Prinzmetal angina, or variant angina, is the most likely diagnosis in this patient. Prinzmetal angina is characterized by spontaneous episodes of angina in association with ST-segment elevation on ECG. The cause is a transient, abrupt, and marked reduction in the luminal diameter of an epicardial coronary artery due to spasm which leads to transient myocardial ischemia. Unlike most other causes of chest pain and ST-segment elevation, the ST-segment returns to baseline rapidly upon resolution of symptoms. This is a diagnosis of exclusion. Patients with variant angina are often younger and exhibit fewer classic cardiovascular risk factors, with the exception of cigarette smoking. Variant angina may be associated with other vasospastic disorders such as Raynaud's phenomenon and migraine headache or its treatment, such as sumitriptan. A history of drug abuse, such as cocaine, may also be present. Costochondritis (A) is musculoskeletal chest wall pain that causes more diffuse pain, in which multiple areas of tenderness are found and can usually be reproduced with palpation. Panic disorder (B) is a particularly common cause of chest pain; however it is not associated with ECG changes and is less consistent with this clinical picture. Unstable angina (D) is a presentation of acute coronary syndrome characterized by new, worsening or persistent chest pain, but in the absence of positive cardiac biomarkers. This can be difficult to distinguish from Prinzmetal angina; however, it would be less likely in a younger patient and it is not associated with other vasospastic disorders such as migraine headaches or Raynaud's phenomenon.

A 72-year-old woman was hospitalized for atrial fibrillation with rapid ventricular rate. She is being discharged today on amiodarone for long term control of her dysrhythmia. Which of the following diagnostic studies should be followed as an outpatient? Coagulation studies Complete blood counts Pulmonary function tests Renal panels

Correct Answer ( C ) Explanation: Pulmonary function tests should be monitored for patients on chronic amiodarone therapy. Amiodarone is a class III anti-arrhythmic drug used to treat many common dysrhythmias. Baseline pulmonary function tests and a chest radiograph with annual pulmonary function tests thereafter are recommended. Pulmonary toxicity generally correlates more closely with the total cumulative dose than the serum levels. It usually occurs months to years after initiation. Several types of pulmonary toxicity may result from chronic amiodarone therapy; however, the most common is a chronic interstitial pneumonitis. A non-productive cough and dyspnea are present in the majority of affected individuals. If signs or symptoms develop the patient should discontinue amiodarone immediately. Other complications of chronic amiodarone therapy include, but are not limited to, thyroid dysfunction, both hypo- and hyperthyroidism, symptomatic hepatitis, corneal microdeposits, optic neuropathy, or dermatologic manifestation such as photosensitivity and blue-gray skin discoloration. Thyroid function tests and hepatic panels are recommended every six months. Dermatologic physical exams and ophthalmologic eye evaluations are recommended as needed for signs or symptoms. These complications are treated by either reducing the dose and discontinuation. Coagulation studies (A), complete blood counts (B) and renal panels (D) are not routinely monitored with amiodarone therapy; however, these studies are important for the monitoring of many different drugs. A few examples include coagulation studies for patients on warfarin, complete blood counts for patients on the antipsychotic, clozapine, and renal panels for patients on angiotensin converting enzyme inhibitors. These are just a few examples.

What is the most common carrier of rabies in the United States? Bats Cats Raccoons Squirrels

Correct Answer ( C ) Explanation: Rabies is a viral disease transmitted by the bite of a rabid animal that results in encephalopathy and near-certain death. While any mammal can carry rabies, in the United States more than 90% of all cases occur in wild animals. Raccoons are the most common carrier, followed by bats, skunks, and foxes. Worldwide, dogs are the most commonly infected animal. Mice, rats, squirrels, and rabbits are unlikely to be carriers and there has never been a known case of rabies transmitted to a human from one of these animals. Bats (A) are the second most common carrier of rabies in the United States. While domestic animals account for less than 10% of carriers in the United States, cats (B) are over twice as likely to be carriers than dogs. Squirrels (D) are unlikely to be carriers of the disease.

An 85-year-old woman presents with generalized weakness for five days. Her vital signs are unremarkable except for a HR of 130 bpm. Her ECG is shown above. Which of the following represents the first-line treatment for this patient? Adenosine Amiodarone Diltiazem Synchronized cardioversion

Correct Answer ( C ) Explanation: The patient has atrial fibrillation with rapid ventricular response. Treatment of atrial fibrillation starts by distinguishing it from other chaotic rhythms (e.g., multifocal atrial tachycardia) and recognition of any underlying causes. Asymptomatic atrial fibrillation at a rate of <100 bpm requires no specific emergent therapy. Stable patients who have a rapid ventricular response (>100 bpm) should receive rate or rhythm control. Because the onset of atrial fibrillation in this patient is unknown, rhythm control in the ED is contraindicated. Intravenous calcium channel blockers (diltiazem, verapamil) or beta-adrenergic blockers (metoprolol) are first-line rate-controlling agents for stable atrial fibrillation. Adenosine (A) is a short-acting AV-nodal blocking agent used in the treatment of AV-nodal re-entry tachycardia (AVNRT), not atrial fibrillation. Amiodarone (B) is a class III antidysrhythmic that is used for the treatment of both ventricular and supraventricular dysrhythmias. It has the ability to cardiovert an irregular rhythm (atrial fibrillation) and should be avoided in this patient. In addition, amiodarone has an unfavorable side effect profile compared to calcium channel blockers. Synchronized cardioversion (D) should not be performed for atrial fibrillation when the exact onset is unknown. Patients should undergo an echocardiogram to evaluate for an atrial thrombus prior to cardioversion. However, in the unstable patient, the benefit of cardioversion outweighs the risk of thromboembolism, and cardioversion is indicated.

A 17-year-old girl presents to your office with a complaint of worsening headaches. The headaches occur most days of the week, worsen with activity, and are sometimes associated with vomiting. She denies a family history of headaches. She is taking ibuprofen for her headaches and doxycycline for acne. Exam is significant for papilledema. A brain MRI is normal, and on lumbar tap the opening pressure is elevated. What is the best next step in treatment? Start acetazolamide Start rizatriptan Stop doxycycline Stop ibuprofen

Correct Answer ( C ) Explanation: The patient has idiopathic intracranial hypertension (formally known as pseudotumor cerebri) as evidenced by papilledema, normal imaging and elevated opening CSF pressure. Idiopathic intracranial hypertension should be suspected in any older child or adolescent with headaches and papilledema. Patients typically describe diffuse headaches that may worsen on straining. It is often associated with visual loss or diplopia, transient visual obscurations, pulsatile tinnitus, limited abduction of one or both eyes, and rarely a decreased level of consciousness. The disorder typically affects obese, hirsute-appearing young women. Idiopathic intracranial hypertension has many drug-induced etiologies as well, including tetracyclines, oral contraceptives, sulfonamides, hypervitaminosis A, phenytoin, corticosteroids and nitrofurantoin. It is appropriate to stop doxycycline to determine if that is the underlying cause of her headaches. Acetazolamide (A) is a treatment option for idiopathic intracranial hypertension, but underlying etiologies, such as doxycycline use, should be excluded first. Triptans (B) are useful as an abortive medication in migraine headaches, but this individual's findings are inconsistent with migranes. Overuse of NSAIDs (D) can be associated with headaches, but highly unlikely in this individual given the findings of increased intracranial pressure.

A 12-year-old diabetic female presents to the ED complaining of multiple episodes of vomiting associated with persistent nausea and generalized abdominal pain. She also complains of fever, chills, and dysuria. Vital signs include a BP of 94/60 mm Hg, RR of 26 breaths per minute, HR of 106 beats per minute, T of 38.3°C, and oxygen saturation of 98% on room air. Her mucous membranes are dry, and her abdomen is tender to palpation. A point-of-care capillary glucose machine reads "high." She is given a 2 L bolus of normal saline. Urinalysis is positive for ketones, glucose, nitrite, leukocyte esterase with 20-30 WBCs. You suspect this patient is in diabetic ketoacidosis secondary to a urinary tract infection. After the normal saline bolus, you start her on an insulin drip at 5 U/hr and administer antibiotics for the UTI. Below are the most recent lab values. What is the most appropriate next step in management? Administer ampule of NaHCO3 with a 0.9% NaCl bolus and increase insulin drip dose Change fluids to 5% dextrose in 0.45% NaCl and increase insulin drip dose Change fluids to 5% dextrose in 0.45% NaCl and maintain insulin drip dose Change fluids to 5% dextrose in 0.45% NaCl and reduce insulin drip dose

Correct Answer ( C ) Explanation: This patient has diabetic ketoacidosis (DKA). Common causes of DKA include a lack of insulin, infection, ischemia, and illicit drug use. The treatment of DKA is multifactorial and takes into consideration electrolyte levels and fluid status. Patients in DKA routinely have a fluid deficit of 100 mL/kg or 5-10 L. Normal saline is the most frequently recommended fluid for initial volume repletion. After 2-4 L of normal saline, many clinicians recommend continuing hydration with 0.45% saline. When the blood glucose level is approximately 250 mg/dL, the fluid should be changed to 5% dextrose in 0.45% saline to avoid iatrogenic hypoglycemia and cerebral edema. In addition to IV fluids, an insulin infusion should be started at 0.1 units/kg/hour. The insulin infusion should continue until ketonemia has cleared and the anion gap has normalized. Although intermittent insulin boluses can also be used, they have not been shown to improve patient outcome and may increase the risk of hypoglycemia. Patients in DKA usually present with profound total-body potassium deficits. This deficit is due to insulin deficiency, metabolic acidosis, osmotic diuresis, and vomiting. The initial serum potassium concentration is usually normal or high because of shift of intracellular potassium to extracellular fluid. If the serum potassium concentration is initially low however, a severe total body potassium deficit is present. Because insulin will further decrease potassium levels, initiation of an infusion should be delayed until serum potassium content can be confirmed, with concurrent repletion if indicated. The patient's anion gap and ketonuria are improving, so the current insulin infusion (B) is adequate. The insulin dose does not need to be lowered (D) in this patient as long as her fluid is switched to 5% dextrose. Routine use of supplemental bicarbonate (A) in the treatment of DKA is not recommended. Risks of bicarbonate use in DKA involves worsened hypokalemia, paradoxical CNS acidosis, worsened intracellular acidosis, impaired oxyhemoglobin dissociation, hypertonicity and sodium overload, delayed recovery from ketosis, elevated lactate levels, and cerebral edema.

A 21-year-old woman presents with shortness of breath, rash and nausea after an insect bite. Her vitals are T 97.7°F, HR 128, BP 85/56, RR 28, oxygen saturation 93%. Exam reveals diffuse hives and posterior pharyngeal swelling. Which of the following should be immediately administered? Epinephrine 1:10,000, 0.3 mL IM Epinephrine 1:10,000, 10 mL IV Epinephrine 1:1000, 0.3 mL IM Epinephrine 1:1000, 0.3 mL IV

Correct Answer ( C ) Explanation: This patient presents with anaphylactic shock from an insect bite and requires immediate administration of epinephrine. Epinephrine is potentially life-saving in severe anaphylactic reactions. During an anaphylactic reaction, mast cells degranulate leading to release of histamines and other immune mediators. These mediators lead to the hallmark symptoms of allergic reactions including hives, nausea and vomiting, airway edema, bronchoconstriction and hypotension. Epinephrine acts immediately on adrenergic receptors to reverse these symptoms. Inhaled beta agonists can also be given to rapidly reverse bronchoconstriction. Many of the other treatments in anaphylaxis have a delayed onset of action. Diphenhydramine (H1 receptor blocker) acts by blocking the effect of histamine on H1 receptors mainly in the skin. Ranitidine, famotidine and other H2 blockers can mitigate some of the gastrointestinal symptoms caused by histamine as well as some of the cutaneous manifestations. Steroids mainly act by stabilizing mast cells from further degranulation but this effect is usually delayed 4-6 hours after administration. Steroids also increase the expression of beta-receptors in the lung increasing the efficacy of inhaled beta-agonists. Epinephrine should be given as 300 - 500 mcg IM in the anterolateral thigh. This dose equates to 0.3 - 0.5 mL of the 1:1000 preparation. Giving epinephrine 1:1000, 0.3 mL IV (D) is a massive overdose and can cause dysrhythmias and cardiac ischemia. Cardiac epinephrine (A & B) (1:10,000 concentration) comes in vials of 10 mL for a total of 1000 mcg of epinephrine. This preparation of epinephrine is reserved for use in patients with cardiac arrest (ventricular fibrillation, pulseless electrical activity or asystole) and should not be given to patients with a blood pressure. 0.3 mL of cardiac epinephrine IM is a 10-fold underdose as this amount only contains 30 mcg of epinephrine.

A 21-year-old man presents with a headache. What feature should raise the concern for a subarachnoid hemorrhage? Age <40 Fever History of intravenous drug use Sudden onset of headache

Correct Answer ( D ) Explanation: A sudden onset of headache should raise concern for subarachnoid hemorrhage (SAH). SAH is a life-threatening disorder that involves extravasation of blood into the subarachnoid space. It accounts for 10% of all strokes. SAH can either be traumatic or spontaneous. Spontaneous SAH is associated with intracranial aneurysms about 80% of the time. Patients typically present with a sudden onset of severe headache that is classically described as "the worst headache of their life." The headache of SAH is often associated with nausea and vomiting (75%) and may have concomitant neck stiffness (25%) and seizures (17%). Meningismsus is present in half of patients. The workup of a patient with a suspicion of a subarachnoid hemorrhage typically involves a noncontrast head CT followed by a lumbar puncture if imaging is unremarkable. Although the ability of CT to detect SAH in patients presenting early in the course of disease has improved greatly over the last decade, lumbar puncture is still recommended to rule out the disorder. Age >40 (A) is associated with a higher risk of SAH. Fever (B) can be seen in SAH, but is unusual. A history of intravenous drug use (C) does not increase the risk of SAH.

A five-year-old boy presents for a well child check. He has had several episodes of acute otitis media during the past 12 months but has otherwise been healthy. His physical exam is notable for weight and length at the 60th percentile. On examination of his right tympanic membrane, there is a yellow mass noted behind the tympanic membrane. There is retraction of the tympanic membrane, without any notable drainage. His mother reports that the ear has drained purulent fluid in the past. The left tympanic membrane is unremarkable. Which of the following is the next best step? Amoxicillin 90 mg/kg/day for 10 days Ciprofloxacin-dexamethasone drops for seven days Refer to audiology Refer to ENT

Correct Answer ( D ) Explanation: Cholesteatoma is a growth in the middle ear, which may be congenital or acquired. Acquired cholesteatoma forms due to chronic negative pressure in the middle ear and is usually associated with chronic middle ear effusions. The negative pressure creates a pouch that traps desquamating squamous epithelial cells and keratin, which accumulates into a mass. Other risk factors include a history of rupture of the tympanic membrane, or a history of tympanostomy tubes, which allow for epithelial cells to implant in the middle ear cavity. The average age of presentation is five years. Typical symptoms include chronic ear drainage, and on examination there may be a yellow or white mass behind the tympanic membrane. The metabolically active squamous cells can invade nearby bony structures such as the ossicles or the temporal bone, allowing the mass to expand into the mastoid cavity or even intracranially. Mastoid involvement is present in about 25 percent of cases. Other complications include permanent hearing loss, loss of cranial nerve VII function, damage to the labyrinthine structures leading to poor balance, or intracranial involvement. Treatment is urgent tympanomastoid surgery by ENT due to the high rate of complications of cholesteatoma. Oral high-dose amoxicillin (A) is the primary therapy for treatment of acute otitis media. This patient does not have AOM or signs of acute infection, so antibiotics would be inappropriate. Otic drops (B) can be used to treat otitis externa. However, this patient does not have signs of inflammation or infection of the external auditory canal. Audiology referral (C) is important for patients with suspected hearing loss, or in patients at risk for hearing loss. Up to 25 percent of patients with cholesteatoma may have associated conductive hearing loss. However, the more urgent step in management is surgical resection of the mass.

A 4-year-old Caucasian boy presents to your office with watery nonbloody diarrhea for the last 3 months. He has occasional abdominal pain prior to the onset of the diarrhea and his mother noted worsening flatulence, especially after meal times. Mom denies any recent travel or camping and the patient does not go to school or daycare. On exam, he is 50% for weight and height, appears well, and has a normal abdominal exam. What is the most likely diagnosis? Celiac disease Inflammatory bowel disease Irritable bowel syndrome Lactase deficiency

Correct Answer ( D ) Explanation: Congenital lactase deficiency is rare and is associated with symptoms occurring on exposure to milk. However, fewer than 50 cases have been reported worldwide. Secondary lactose intolerance follows small bowel mucosal damage from conditions such as celiac disease and rotavirus infection and is usually transient, improving with mucosal healing. Patients can develop diarrhea, abdominal cramps, abdominal pain, and flatus following ingestion of milk products. Symptom severity depends on the amount of lactose ingested and the fat content of the product (skim milk empties from the stomach faster causing more symptoms). It is not associated with a failure to thrive. Lactase deficiency can be diagnosed by H2-breath test or by measurement of lactase activity in mucosa tissue retrieved by small bowel biopsy. Diagnostic testing is not mandatory, and often simple dietary changes that reduce or eliminate lactose from the diet relieve symptoms. Treatment of lactase deficiency consists of a milk-free diet. A lactose-free formula (based on either soy or cow's milk) can be used in infants. In older children, low-lactose milk can be consumed. Addition of lactase to dairy products usually lessens the symptoms. Inflammatory bowel disease (B) (IBD) is characterized by bloody stools and failure to thrive. Irritable bowel syndrome (C) (IBS) is characterized by diarrhea that alternates with constipation. The associated abdominal pain is often relieved by defecation and there are no systemic features. Celiac disease (A) is associated with diarrhea, abdominal pain and distension, but is usually associated with a failure to thrive. Moreover, the condition is caused by a permanent gluten intolerance.

Which of the following is characteristic of Sjögren's syndrome? Granuloma formation with necrotizing vasculitis of the small arteries and veins Infection of the upper respiratory tract with group A streptococci Inflammation of the medium and large-sized arteries Lymphocytic infiltration of the exocrine glands

Correct Answer ( D ) Explanation: Sjögren's syndrome is an autoimmune disease characterized by lymphocytic infiltration of the exocrine glands. This infiltration results in xerostomia (dry mouth) and dry eyes. Sjögren's syndrome can present as a primary disorder or as secondary to other autoimmune rheumatic diseases such as rheumatoid arthritis, scleroderma or systemic lupus erythematosus. Treatment is aimed at relief of symptoms and limiting the damage caused by chronic xerostomia and dry eyes. Other autoimmune diseases have very distinct presentations. Granuloma formation with necrotizing vasculitis of small arteries and veins (A) is seen in the histology of Wegener's granulomatosis (Granulomatosis with polyangiitis). Acute rheumatic fever is caused by infection of the upper respiratory tract with group A streptococci (B). Inflammation of medium and large-sized arteries (C) is characteristic of giant cell arteritis. These diseases all differ from Sjögren's syndrome in that there is no involvement with the exocrine glands.

A 45-year-old man presents after swallowing a pen. A review of his previous visits shows two prior evaluations for the same thing. He reports this is because of post-traumatic stress disorder. An X-ray confirms the presence of a pen in the esophagus. What is his most likely diagnosis? Borderline personality disorder Drug-seeking behavior Malingering Munchausen syndrome

Correct Answer ( D ) Explanation: Munchausen syndrome is a form of factitious disorder in which patients feign illness for some sort of personal gain. Factitious disorders are different from somatic disorders and malingering because of intention. Patients will often have objective physical findings as a result of their self-harm. In some cases, they may induce a serious illness in order to have prolonged hospitalization in the patient role. They are willing to undergo procedures in order to maintain their sick role. Borderline personality disorder (A) is characterized by a pattern of instability in interpersonal relationships and self-image. Patients have extremely labile moods and are at risk for suicide, eating disorders, post-traumatic stress disorder and substance abuse. Patients classically split providers identifying some as good and some as bad subsequently pinning them against each other in the course of their care. Drug-seeking behavior (B) is a challenging diagnosis in which a patient feigns illness as a type of malingering in order to obtain drugs. Typically the patient has active issues with substance abuse. Malingering (C) is separate from somatization disorders and factitious disorders because the patient simulates a disease for some secondary gain. Factitious patients will do things creating actual objective findings.

A 16-year-old girl presents to clinic with 3 weeks of worsening right knee pain. She attends dance class since age 5 and now dances 20 hours a week. She denies any inciting injury and continues to dance on the injured leg. On exam, she has pain at the inferior and medial pole of the right patella with no swelling or erythema of the knee. Which of the following is the most likely diagnosis? Anterior cruciate ligament rupture Medial meniscus tear Patella dislocation Patellofemoral pain syndrome

Correct Answer ( D ) Explanation: Patellofemoral pain syndrome occurs in about a quarter of those involved in athletics and more commonly in women and those between the ages of 10 and 35 years. Symptoms include pain with going up and down stairs and prolonged sitting or squatting. Individuals may have a sensation of the knee buckling or giving way. Swelling, popping or grinding sensations may be present. Treatments include non-steroidal anti-inflammatory medications, ice, quadriceps strengthening, stretching, patella bracing and orthotics. The anterior cruciate ligament (A) is usually injured by hyperextension, rapid deceleration or twisting movement and is characterized by early swelling and immediate severe pain. A meniscus tear (B) in young adults and adolescents usually occurs from a sudden flexing or twisting of the knee. A loose piece of cartilage can get stuck in the joint and cause temporary locking of the knee and prevention of full extension of the leg. Patella dislocation (C) is usually caused by sudden twisting of the leg or a direct blow that causes the patella to dislocate lateral of its normal placement in the patellofemoral groove. The patient experiences intense pain and typically has effusion on exam.

A 60-year-old woman with a history of congestive heart failure presents with substernal chest pain that awoke her from her sleep. She has dysphagia when swallowing solids and odynophagia when swallowing both solids and liquids. She reports a history of chest pain in the past but states that this pain is different in that it is sharp and occurs only after swallowing. Her medications include metoprolol succinate, furosemide, potassium chloride, simvastatin, isosorbide dinitrate, and levothyroxine. Which of the following is most likely to lead to the correct diagnosis? Barium esophagram CT angiogram of the chest Electrocardiogram Questioning her about her medication use

Correct Answer ( D ) Explanation: Pill esophagitis is caused by a pill becoming trapped in a collapsed esophagus. Questioning the patient about when and how she takes her medication may elicit information suggesting pill esophagitis is the cause of her chest pain and would be the next step in determining a diagnosis. The position of the patient, size of the medication, and the amount of fluid ingested with the medication are important determinants of the risk of medication-induced esophagitis. Ingestion of a pill immediately prior to sleep is also associated with an increased risk of medication-induced esophagitis as both salivation and swallowing frequency are markedly reduced during sleep. Patients with medication-induced esophagitis usually present with retrosternal pain or heartburn, odynophagia, and dysphagia. In some cases, the pain may be so severe that swallowing saliva is difficult. Rarely, patients may have hematemesis, abdominal pain, and weight loss. Many medications can cause pill esophagitis, but NSAIDs, potassium chloride, iron, vitamin C, bisphosphonates, and antibiotics are most common. The onset of the symptoms can occur within a few hours to one month after ingestion of the culprit medication. Patients often have a history of swallowing a pill without water, commonly at bedtime. An upper endoscopy should be performed in patients with symptoms that are severe; infrequently seen (hematemesis, abdominal pain, and weight loss); or persist for one week after discontinuation of the culprit medication. In such cases, an upper endoscopy with biopsy serves to establish the diagnosis and to rule out other etiologies. The most important aspect of management of medication-induced esophagitis is to avoid further esophageal injury. Culprit medications should be stopped, if possible, or replaced with liquid formulations. Acid suppression medications can be used for symptomatic relief. The differential diagnosis for acute chest pain is long and includes many life-threatening pathologies. If there is concern for acute coronary syndrome being the cause of her chest pain then ECG (C) is important to obtain. However, it is not helpful in the diagnosis of pill esophagitis. CT angiogram of the chest (B) would be helpful in diagnosing acute aortic dissection, aortic aneurysm, or pulmonary embolism as a cause of her chest pain, but would provide little information about this patient's current pathology. A barium esophagram (A) can be used to diagnose esophageal stricture and perforation, however, its use is limited in the diagnosis of esophagitis.

A 26-year-old obese woman with a 2-year history of increased acne, abnormal hair growth, and menstrual abnormalities presents to her obstetrician for an infertility workup. A pelvic ultrasound reveals enlarged cystic ovaries. She desires to become pregnant. Which of the following is the first line treatment? Clomiphene citrate Gonadotropin therapy Spironolactone Weight reduction

Correct Answer ( D ) Explanation: Polycystic ovarian syndrome (PCOS) is characterized by androgen excess, insulin resistance, and gonadotropin abnormalities. PCOS is the most frequent cause of anovulatory infertility. The diagnostic criteria for PCOS are chronic anovulation and hyperandrogenism (established by hormone measurements or clinical findings such as acne or hirsutism) in women whom secondary causes have been excluded. Polycystic appearing ovaries on ultrasound examination are a nonspecific finding in PCOS. Important factors in the physical examination include the following: the presence of acne, balding, or clitoromegaly; the distribution of body hair; enlargement of the ovaries (based on a pelvic examination); and signs of insulin resistance. Weight reduction is the first line treatment for polycystic ovarian syndrome for women who desire to conceive. As little as 5% total weight reduction has been shown to improve the metabolic and reproductive abnormalities in PCOS. Clomiphene citrate (A) is the next line of treatment if weight loss does not lead to return of ovulation. Multiple studies have shown conflicting evidence comparing the efficacy of metformin and clomiphene for ovulation induction, however clomiphene remains the first line pharmacologic agent for ovulation induction in PCOS. Gonadotropin therapy (B) is used if clomiphene citrate fails. Low-dose gonadotropin therapy is recommended over high-dose therapy. Spironolactone (C) is the first-line agent to treat hirsutism in women with PCOS and has no role in ovulation induction. Combination oral contraceptives are also used to prevent hirsutism and menstrual abnormalities in patients whom do not wish to conceive, however they are not first-line.

A 46-year-old man presents with left eye redness since this morning as seen in the image above. He denies any trauma, pain, discharge, or vision changes. Which of the following is the most likely diagnosis? Conjunctivitis Hyphema Retrobulbar hematoma Spontaneous subconjunctival hemorrhage

Correct Answer ( D ) Explanation: Spontaneous subconjunctival hemorrhage is caused by the rupture of a small subconjunctival vessel that leads to bleeding between the conjunctiva and the sclera. Symptoms include a painless, reddened eye without visual changes. Hemorrhage can occur spontaneously or secondary to forceful coughing, sneezing, or vomiting. Patients using anticoagulants may be at greater risk. Treatment is observation and reassurance, although, sometimes artificial tears may be used to soothe irritated or chemotic tissue. Conjunctivitis (A) is an inflammation of the conjunctiva caused by either a viral or bacterial infection, or an allergen, irritant, or chemical exposure. It is characterized by pain, foreign body or scratchy sensation, watery or purulent discharge, and conjunctival hyperemia. Hyphema (B) is characterized by bleeding into the anterior chamber of the eye usually secondary to blunt trauma. Symptoms of hyphema include eye pain and a sudden decrease in visual acuity that may gradually worsen. Retrobulbar hematoma (C) is usually the result of orbital trauma. Increased pressure on the optic nerve can result in blindness. The condition is associated with proptosis and restriction of extraocular movements.

A patient presents with ankle pain after twisting it while walking. Which of the following clinical findings necessitates an ankle X-ray? Age >50 Tenderness to the anterior edge of the lateral malleolus Tenderness to the anterior edge of the medial malleolus Tenderness to the posterior edge of the lateral malleolus

Correct Answer ( D ) Explanation: The patient presents with an injury suspicious for an ankle sprain and the decision to perform an X-ray can be guided by the Ottawa Ankle Rule. Ankle sprains refer to ligamentous injury to the ankle. In patients presenting with ankle pain after trauma, it is important to determine the presence or absence of a fracture as this may guide management. The Ottawa Ankle Rule was developed to reduce the number of X-rays for ankle injuries. Studies have found it to have a sensitivity near 100%. Application reduces the use of X-ray by 36%. Below are the components of the rule. Therefore, tenderness to the posterior edge of the lateral malleolus necessitates an X-ray. There is no specific age cutoff (A) for the Ottawa Ankle Rules and it has been successfully applied in children over the age of 6. Tenderness anterior to the lateral malleolus (B) is not a criterion for an X-ray and this is commonly seen in patients who sprain the anterior talofibular ligament (ATFL). Tenderness to the anterior edge of the medial malleolus (C) is also commonly seen in ankle sprains.

This is an audio question. Please hit play in the file above. What is the most likely diagnosis? Chlamydophila pneumonia Cough-variant asthma Croup Pertussis

Correct Answer ( D ) Explanation: This is the classic unremitting paroxysmal cough followed by a "whoop" that is characteristic of pertussis. There may be as many as 40 episodes per day and symptoms can last up to four weeks. The characteristic whoop may be absent in adults and in infants less than six months old. Chlamydophila (formerly Chlamydia) pneumonia (A) is a common cause of atypical pneumonia in young adults. In infants, a staccato-like cough may be present but not the characteristic inspiratory whoop seen with pertussis. Cough-variant asthma (B) is associated with paroxysmal coughing and post-tussive vomiting but does not share the same frequency as the cough of pertussis or the characteristic whoop. Croup (C) is a viral infection that has a characteristic barking cough that sounds like a seal. Patients may also exhibit inspiratory stridor in more severe illness.

A 23-year-old man with a history of opiate abuse presents with tachycardia, hypertension and mydriasis. Which of the following is true regarding management? Antiemetics should be witheld as they may cause serious side effects Consider ICU admission for the potential for symptoms to worsen Naloxone should be given Symptom control may be achieved with clonidine

Correct Answer ( D ) Explanation: This patient presents with opiate withdrawal and management should focus on the relief of symptoms. The onset of withdrawal symptoms depends on the opiate agent used. Heroin withdrawal typically occurs within 4-6 hours of discontinuation whereas methadone has a longer half-life and withdrawal may be delayed 24-48 hours. Withdrawal leads to sympathetic discharge and adrenergic hyperactivity. Symptoms include CNS excitation, tachypnea and mydriasis. Tachycardia and hypertension are common. Additionally, patients will often complain of nausea, vomiting and diarrhea. Physical examination may also reveal piloerection, yawning, rhinorrhea and lacrimation. Opiate withdrawal is not life-threatening. Supportive and symptomatic care can decrease the patient's discomfort. Clonidine has been used to mitigate symptoms as have antiemetics. Antiemetics (A) can safely be used to control nausea and vomiting associated with opiate withdrawal. ICU admission (B) is unnecessary as opiate withdrawal is not life-threatening. Naloxone (C) is used to reverse the effects of opiates.

A 33-year-old man with schizophrenia treated with haloperidol presents with involuntary, writhing movements of the tongue and face. Additionally, the patient has choreoathetoid movements of the trunk and arms. What management is indicated? Benztropine Diphenhydramine Lorazepam Stop haloperidol

Correct Answer ( D ) Explanation: This patient presents with tardive dyskinesia (TD) and should have his haloperidol therapy stopped. TD is characterized by involuntary movements, particularly of the face and tongue that occur after years of treatment with neuroleptic agents. The movements are described as grimacing, writhing and choreathetoid in nature. Duration of treatment, cumulative dose, patient age and the presence of preexisting brain damage are directly related to the incidence of TD. In patients with mild symptoms, cessation of the neuroleptic drug or decreasing the dose may result in resolution of symptoms. Patients with moderate to severe symptoms are harder to treat. Reserpine and tetrabenazine have shown some benefit in combination with stopping the offending agent. Benztropine (A) and diphenhydramine (B) are effective in treating acute dystonic reactions and akathesia but are not effective in treating TD. Lorazepam (C) will lead to sedation of the patient but will not treat TD.

A 35-year-old woman comes to the urgent care clinic complaining of a 3-week history of difficulty seeing, blurred vision, eye pain, and cough. She describes the cough as being dry and nonproductive. She has no past medical history and takes no medications. Her blood pressure is 140/85 mm Hg and her temperature is 38.1°C (100.5°F). Ophthalmologic examination shows uveitis. Chest X-ray shows bilateral hilar adenopathy. Which of the following additional findings would also most likely be found in this patient? Ferruginous bodies Increased sweat chloride Positive PPD skin test Subcutaneous nodules

Correct Answer ( D ) Explanation: Uveitis, bilateral hilar adenopathy, and a dry cough most likely indicate sarcoidosis. Sarcoidosis is granulomatous disease that can form nodules in multiple organs. When these nodules form in the skin, they are referred to as subcutaneous nodules (ie, erythema nodosum). Patients with sarcoidosis typically present with fatigue, weight loss, arthritis, dry eyes, blurry vision, and respiratory symptoms (eg, cough, dyspnea). Management usually involves the use of corticosteroids (eg, prednisone). Ferruginous bodies (A) are classically associated with interstitial lung disease, specifically asbestosis. They are fibers of asbestos coated with an iron-rich material derived from ferritin. Increased sweat chloride (B) is typically seen in patients with cystic fibrosis. This results from impaired chloride transport due to a mutation in the CFTR gene. A positive PPD skin test (C) would be seen in a person exposed to tuberculosis.

An obese 45-year-old man works for the local electric company as a utility-pole lineman. He presents with painful, tortuous superficial veins on his lower legs. These areas are not associated with erythema, warmth or skin breaks. Which of the following initial plans is most appropriate at this time? Dangle your legs at night over the side of your bed for 30 minutes Increase your daily exercise routine Schedule an appointment with a vascular surgeon Use compressive stockings during the day

Correct Answer ( D ) Explanation: Venous insufficiency is mainly a chronic condition in which the "musculovenous pump" of the distal extremities does not properly return deoxygenated blood the heart, resulting in retrograde flow and distal blood back-up. A major cause of this is venous valvular incompetence, in which the vein leaflets do not operate properly. There are many causes of valvular incompetence, the most common of which is repeated vein injury, which can occur in utility pole lineman, or prior deep vein thrombosis. Two complications of valvular incompetence are varicose veins (with or without symptoms) and thrombophlebitis (inflamed superficial and/or deep vein inflammation due to a blood clot). Varicose veins are enlarged and twisted veins which have been dilated due to incompetent valves, retrograde blood flow and back-up of venous blood. Varicose veins most commonly occur in the superficial veins of the legs, especially when standing. They can be asymptomatic but have unsightly cosmesis, or they can be painful and be associated with distal edema, venous eczema, lipodermatosclerosis (skin thickening) and ulceration. They are more common in women, and are associated with pregnancy, obesity, aging, menopause, abdominal straining and prolonged standing. A complication is progression into thrombophlebitis. Treatment includes leg elevation, graduated compression stockings with or without intermittent pneumatic compression, aspirin or NSAIDs and topical anti-inflammatory gels. Leg elevation (A), especially at night, is recommended. Although somewhat counterintuitive, there is no supporting evidence to recommend regular exercise (B) as a viable treatment of symptomatic varicose veins. There are several surgical methods used in treating symptomatic varicose veins. However, this patient has not tried any conservative measures yet, and as such, referral to a surgeon (C) is not recommended at this time.

Which of the following patients is at greatest risk of developing West Nile meningoencephalitis? 22-year-old man status postrenal transplant 3-year-old girl who is unvaccinated 58-year-old man with diabetes and hypertension 82-year-old woman with dementia

Correct Answer ( D ) Explanation: West Nile virus is a zoonotic infection that first appeared in the U.S. along the eastern seaboard in 1999 but can now be found nationwide. West Nile encephalitis (WNE) is endemic in the Middle East, Africa, and Asia. Birds serve as the primary host, and it is transmitted by the bite of a mosquito. WNE usually occurs in the summer, when mosquitoes, wild migratory birds, and humans are in close proximity outdoors. Most people infected with West Nile virus are asymptomatic. When present, symptoms are typically mild and include fever, headache, and fatigue. Severe disease, however, can cause central nervous system manifestations including meningitis, encephalitis, and myelitis. The biggest risk factor, by far, for neuroinvasive West Nile disease is advanced age. There are a handful of confirmed cases of transmission via organ transplantation or blood transfusion. Although immunosuppression (A and C) puts people at increased risk and invasive disease in young children can be more severe (B), West Nile meningoencephalitis is far more common in the geriatric population than in any other group.


Ensembles d'études connexes

Anatomy: Heart anatomy and physiology

View Set

Hunter's Ed - Unit 2/ parts 1 and 2

View Set

OCE1001 FSU Final Exam All Chapters

View Set

ECON 136 Business Strategies Final MC

View Set